IM/GI → Cholecystitis, Cholangitis, Pancreatitis, PUD, mesenteric ischemia, re-feeding syndrome, Gastric cancer,

¡Supera tus tareas y exámenes ahora con Quizwiz!

Stool analysis for Ova & parasites - Pt likely presents w/a parasitic infection - Most likely d/t HOOKWORM (Ancylostoma duodenale/ Necator Americanus) -A) Signs/Symptoms -----1) Pulmonary symptoms → (KEY SYMPTOM FOR HOOKWORM) ---------(Cough, dyspnea, wheezing) -----2) Iron deficiency anemia -----3) Malabsorption symptoms ---------(Flatulence, abd pain, bloating) -----4) Eosinophilia -B) MGMT ----a) Dx tests --------1) Stool for ova & parasites ------------a) Specimens on 3 separate days ----b) TMT --------1) Albendazole & Iron replacement CT scan of abd w/ contrast - Used if suspected Inflammatory bowel disease and/or GI cancer ---[Pts age make him unlikely to have GI cancer] ---[Pt has No IBD/Crohns symptoms]

(UWS2E1) A 16B, presents d/t intermittent nausea, watery diarrhea, & a bloating sensation for the past several months. The pt has also had episodic peri-umbilical pain, especially after a meal, & has lost 5-lbs since the symptoms began. He emigrated from rural Thailand w/ his fam approximately 8 months ago. Shortly after arriving in the US, the pt developed a dry cough, SOB, & wheezing that spontaneously resolved after several days; otherwise, he had no medical conditions. The pt is afebrile, & other Vitals are normal. Bilateral breath sounds are normal. Heart sounds are normal w/ no murmur. The abd is soft & non-tender, & bowel sounds are normal. Labs show: - Hb---------------------------------10.4 - MCV--------------------------------72 - Leukocytes-------------------------8K ---Neutros---------------------------60% ---Eosinos----------------------------10% ---Lymphos---------------------------30% - Albumin-----------------------------3.4 - Amylase-----------------------------91 Which of the following is the most appropriate Dx study for this pt? (CT scan of abd w/ oral contrast VS Stool analysis for ova & parasites)

Intestinal bacterial translocation -Pt presents w/ Spontaneous bacterial peritonitis (SBP) -A) RF ---1) Cirrhosis ------Signs/Symptoms of cirrhosis include ----------Spider angiomas, ascites, thrombocytopenia, coagulopathy -B) Signs/symptoms (of SBP) ---1) Fever (100F) ---2) Diffuse abd pain ---3) Tachycardia ---4) Mental status changes -------(abnormal connect-the-numbers test) ---5) Hypotension ---6) Hypothermia ---7) Paralytic ileus ------a) Dilated loops of bowel on Abd XR -C) MGMT ----a) Dx tests -------1) Paracentesis ------------a) Ascitic fluid neutro count ≥250/mm3 (DIAGNOSTIC) ------------b) ± Gm(-) organisms ----b) TMT --------1) Antibiotics ------------(3rd gen cephalosporins, fluoroquinolones) Decreased renal clearance of neurotoxins -This describes Hyperammonemia -Pt has evidence of renal failure (Cr of 1.5) & this can cause confusion & abd pain - However, key differences include: ---1) These symptoms would be rare in pts w/ BUN < 100 ---2) Pt would have a FLAPPING tremor -------[pt has no tremor]

(UWS2E1) A 43F, presents to the ED d/t 2 days of confusion & lethargy. The pt was Dx'd w/ Hep C infection 10-yrs ago during and eval of elevated liver values but did not follow up afterwards. She drinks a pint of liquor daily & in her 20's used injection drugs. Temp is 100.8F, BP is 127/70, pulse is 110/min, RR is 20/min. The pt has scleral icterus & scattered spider angiomas. The abd is distended w/ shifting dullness on percussion. There is diffuse abd tenderness but no guarding or rebound tenderness. She also has bilateral ankle edema. Labs show: - Hb---------------------------------10.8 - Plts---------------------------------90K - Leukocytes-------------------------11K - Na+---------------------------------130 - BUN---------------------------------26 - Cr-----------------------------------1.5 **********LFT's************ - Albumin----------------------------2.8 - Total bili----------------------------4.3 - ALP---------------------------------120 - AST---------------------------------87 - ALT---------------------------------49 - INR----------------------------------1.6 (N 0.8-1.1) - aPTT---------------------------------25 sec Which of the following is the most likely reason for this pts condition? (Decreased renal clearance of neurotoxins VS Intestinal bacterial translocation)

Endoscopic stent -Palliative TMTs for pancreatic cancer include: ---1) Endoscopic Biliary stent ---2) Permanent ablation of celiac nerves Ursodeoxycholic acid - This TMT is used for INTRAHEPATIC cholestasis - Usually seen in the setting of PREGNANCY ------[Pancreatic adenocarcinoma causes EXTRA-HEPATIC cholestasis d/t obstruction in the head of pancreas (from tumor)]

(UWS2E1) A 63M, presents d/t severe itching that interferes w/ sleep. He has noted yellow discoloration of his eyes & recently his skin. His appetite has decreased & he has lost 15-lbs over the past 3 months. His stools are pale. The pt has a PMHx sig for DM II & HTN. He has smoked 1-pack of ciggs daily for the past 30 years & drinks occasionally. Vitals are normal. Exam shows marked jaundice. Abd CT shows a mass in the head of the pancreas & multiple liver lesions suspicious for METs. Biopsy of the pancreatic lesion shows adenocarcinoma. He is Dx'd w/ non-resectable metastatic pancreatic cancer. Total bilirubin is 15.4. Which of the following would be most effective for palliative symptom control? (Endoscopic stent VS Ursodeoxycholic acid)

Pancreas - Pt most likely has pancreatic cancer -A) Signs/Symptoms ---1) Painless jaundice ------a) In an older pt painless jaundice is CONSIDERED PANCREATIC CANCER UNTIL PROVEN OTHERWISE ---2) Migratory thrombophlebitis ---3) Abd pain -------a) radiates to the back in later stages ---4) Wt loss, anorexia ---5) ± Succussion -B) MGMT ----a) Dx tests -------1) Auscultation -----------a) Succussion splash ---------------Indicates gastric outlet obstruction -------2) CT abdomen ----b) TMT --------1) Whipples -----------a) If tumor is localized & accessible --------2) ERCP ******If disease is too advanced→ Palliative, including: --------3) Biliary Stenting --------4) Permanent ablation of celiac nerves -----------(to reduce back pain) Gallbladder - Can also present w/ painless jaundice & weight loss - However, key differences include: ---1) JAUNDICE PRESENTATION ------a) Pt has Jaundice as first/early symptom -----------[Jaundice is MORE likely to present as FIRST symptom in PANCREATIC cancer] ---2) ETIOLOGY ------Cholangiocarcinoma is MUCH MORE RARE than pancreatic cancer ---3) OTHER KEY SYMPTOM -------a) Cholangiocarcinoma also commonly presents w/ Pruritus -----------[pt has no pruritus]

(UWS2E1) A 64M, presents by his wife b/c she has noticed yellowish discoloration of his eyes & skin for the past 3 months. He has not had any nausea or vomiting, but she has noticed decreased appetite & Abd discomfort recently. Temp is 98.8F, BP is 150/90, & RR is 16/min. PE reveals an underweight white male in no acute distress. There is some fullness in the RUQ. Labs show: - Total bili---------------------------12.5 - AST--------------------------------95 - ALT--------------------------------65 - ALP--------------------------------350 - BUN--------------------------------14 - Cr----------------------------------1.2 Which of the following is the most likely primary site for this pts pathology? (Gallbladder VS Pancreas)

Administer broad-spectrum antibiotics -Pt presents w/ Spontaneous bacterial peritonitis (SBP) -A) Etiology ---1) Cirrhosis ------SBP is a major source of mortality & morbidity in pts w/ cirrhosis ------Signs/Symptoms of cirrhosis include ----------Spider angiomas, ascites, thrombocytopenia, coagulopathy -B) Signs/symptoms (of SBO) ---1) Fever (100F) ---2) Diffuse abd pain ---3) Tachycardia ---4) Mental status changes -------(abnormal connect-the-numbers test) ---5) Hypotension ---6) Hypothermia ---7) Paralytic ileus -C) MGMT ----a) Dx tests -------1) Paracentesis ------------a) Ascitic fluid neutro count ≥250/mm3 (DIAGNOSTIC) ------------b) ± Gm(-) organisms/Gm(+) cocci ----b) TMT --------1) Antibiotics ------------(3rd gen cephalosporins, fluoroquinolones) Administer IV furosemide -Used to manage ascites -However, would NOT BE USED in acutely ill pts that are ALREADY HYPOTENSIVE → b/c they can further lower BP -----[pts HYPOTENSIVE; BP is 108/68]

(UWS2E1) A 54M, presents to the ED d/t 2 episodes of coffee-ground emesis since yesterday. The pt had episodic nausea & abd discomfort for the past several months. He consumes a pint of vodka daily. Upper endoscopy reveals a non-bleeding esophageal varices, diffuse gastric erythema, & a small duodenal ulcer. The pt is treated w/ IV fluids & a PPI. He has no further episodes of hematemesis in the hospital, but the next day he becomes lethargic. BP is 108/68 & pulse is 98/min. The pt requires repeated physical stimuli to remain awake. PE shows muscle wasting, gynecomastia, & several spider angiomas. The abd is distended w/ a protruding umbilicus. He has mild pitting edema of the bilateral LEs. Labs show: - Hct---------------------------------32% - MCV-------------------------------102 - Leukocytes-------------------------8K - Na+---------------------------------136 - K+-----------------------------------3.8 - Cr-----------------------------------0.8 **********LFT's************ - Albumin----------------------------3.1 - Total bili----------------------------2.0 - ALP---------------------------------120 - AST---------------------------------62 - ALT---------------------------------24 - INR----------------------------------1.4 (N 0.8-1.1) Diagnostic paracentesis shows a peritoneal fluid leukocyte Ct of 400/mm3 w/ 90% neutros & an albumin of 1.1. Which of the following is the best next step in MGMT? (Administer broad-spectrum antibiotics VS Administer IV furosemide)

Bloodstream infection -RFs for TPN (Parenteral nutrition) include: ---1) Central-line associated bloodstream infection (GREATEST RISK) ------a) Candida & Coag Neg staph are most common ---2) Volume overload ------a) also a potential complication → however MUCH LESS likely than bloodstream infection ---3) Cholelithiasis ------a) Usually seen in pts w/ TPN for >2 weeks ----------[pt has been just started TPN] Re-feeding syndrome - Can also occur in pts when food is re-introduced -However, key differences include: ---1) Usually occurs in pts that are SEVERELY MALNOURISHED -------[pt is NOT severely malnourished]

(UWS2E1) A 57 previously healthy male has a positive fecal occult blood test & mild iron defiency anemia. Colonoscopy reveals a mass in the ascending colon. Histopahtology is consistent w/ adenocarcinoma. Imaging studies reveal no metastatic disease. The pt undergoes right hemicoloectomy w/ ileocolonic anastomosis. The post-op course is complicated by prolonged ileus, but recovery of the bowel function is expected in 7-10 days. Parenteral nutrition is started thru a central venous catheter. BMI is 26. This pt is at greatest risk for developing which of the following complications of nutritional therapy? (Bloodstream infection VS Re-feeding syndrome VS Cholelithiasis OR Volume-Overload)

GI bleeding - Pt presents w/ signs suggesting hepatic encephalopathy -A) Signs/Symptoms → (for hepatic encephalopathy) ----1) Underlying Cirrhosis ----2) Lethargy & confusion ----3) Asterixis (flapping tremor) -B) Etiology → (for hepatic encephalopathy) includes: ----1) Drugs -------(sedatives/narcotics) ----2) Hypovolemia -------(diarrhea) ----3) Electrolyte changes -------(hypokalemia) ----4) ↑ Nitrogen load --------(GI bleeding) ----------Pt presents w/ signs of GI bleeding including ------------a) Positive occult blood test ------------b) Anemia ------------c) Black stools ------------d) Elevated BUN:Cr ratio (>20:1) → suggests upper GI bleed is absorbed in small intestine) ----5) Infection --------(pneumonia/UTI/SBP) ----6) Portosystemic shunting --------(TIPS) -C) MGMT -----a) Dx tests --------1) Diagnostic paracentesis -----b) TMT --------1) Correct precipitating causes ------------a) i.e., fluids/antibiotics ------------b)↓ Blood ammonia concentrations ----------------(lactulose/rifaximin) SBP - Can also cause hepatic encephalopathy in pts w/ underlying cirrhosis - However, key differences include: ---1) Pt would have diffuse Abd pain -------[pt has NO ABD PAIN] ---2) Pt would have signs of infection (i.e., hypotension, tachycardia, fever) -------[pt has NONE of these symptoms] Dietary changes -Can precipitate HE by causing increased ammonia levels - However, key differences include: ---1) Pt has LIMITED protein intake -------[Pt recently REDUCED all protein in diet] ---2) Generally only seen in pts that have required a trans-jugular intrahepatic portosystemic shunt

(UWS2E1) A 62M, w/a PMHx sig for liver cirrhosis caused by Hep C presents to the ED d/t 2 days of progressive confusion & drowsiness. The pt has had no nausea, vomiting, or abd pain but has had black stools for the past few days. A month ago, he had a similar change in mental status in the setting of SBP; his symptoms improved after antibiotic tmt. The pt takes furosemide & spironolactone daily & occasionally uses acetaminophen for headaches. He does not drink or smoke. His wife says that recently, after reading some info on the internet, the pt started restricting all protein in his diet. Temp is 98.6F, BP is 120/78, & pulse is 84/min. The pt is somnolent & has to be woken up several times to answer questions. The oropharynx is most w/out lesions. Cardiopulmonary exam is unremarkable. The abd is distended w/ moderate ascites. Stool occult blood tests is positive. Bilateral LE pitting edema is present. The pt has asterixis w/out other focal neuro deficits. Labs show: - Hb---------------------------------10 - PLTS-------------------------------115K - Leukocytes------------------------8,2K - Na+---------------------------------131 - K+-----------------------------------4 - HCO3------------------------------28 - BUN--------------------------------32 - Cr----------------------------------1.1 **********LFT's************ - Albumin----------------------------2.5 - INR---------------------------------1.4 (N 0.8-1.1) - Ammonia, plasma-----------------140 (N 40-80) Diagnostic paracentesis shows a cell Ct of 1000/mm3 w/ 15% neutros, a total protein level of 1.8, & an albumin levels of 1. Which of the following is the most likely precipitant of the current episodes of confusion in this pt? (GI bleeding VS SBP VS Dietary changes)

Bile duct obstruction from migrating worms - Pt presents w/ round worms in surgery -A) Complications include: ----1) Obstruction at the ileocecal valve ----2) Retrograde migration w/in the biliary tree -B) Signs/Symptoms ----1) Jaundice -C) Associated labs ----1) DIRECT hyperbilirubinemia Transfusion RXN - Also presents w/ jaundice and occurs in setting of blood transfusion - However, key differences include: ---1) Would present w/ INDIRECT hyperbilirubinemia Resorption of hematoma - Would present w/ INDIRECT hyperbilirubinemia

3 days after an exploratory laparotomy for a stab wound to the abd, a 14y/o boy has jaundice. He recently immigrated to the USA from the Caribbean islands. During the operation, three small enterotomies caused by the knife were repaired. Multiple long tubular structures, thought to be roundworms, were felt w/in the lumen of the bowel; they were not removed. He required 2 units of packed RBC intra-operatively. Exam shows scleral icterus. The abd is mildly distended & non-tender. Labs show: - Hb--------------------------------13.5 - Leukos----------------------------10,5K - Bilirubin total---------------------3.5 --- Direct---------------------------2.9 - ALP--------------------------------210 - AST--------------------------------40 - ALT---------------------------------55 Which of the following is the most likely mechanism of this pts jaundice? (Bile duct obstruction from migrating worms VS Transfusion RXN VS Resorption of hematoma)

Laparoscopic appendectomy - Pt present w/ signs of appendicitis -A) MGMT ----a) Dx tests -------1) Abd US ----------a) Dilated non-compressible appendix ----------b) Echogenic peri-appendiceal fat ----b) TMT -------1) IV fluid resuscitation & IV antibiotics -------2) Laparoscopic Appendectomy Oral Amoxicillin & Clavulanic acid - ORAL antibiotics are NOT INDICATED b/c most pts have some degree of ileus - This makes the ABSORPTION of antibiotics UNRELIABLE

A 19-year-old man comes to the emergency department because of abdominal pain, nausea, and vomiting for 4 hours. Initially, the pain was dull and located diffusely around his umbilicus, but it has now become sharper and moved towards his lower right side. He has no history of serious illness and takes no medications. His temperature is 38.2°C (100.7°F) and blood pressure is 123/80 mm Hg. Physical examination shows severe right lower quadrant tenderness without rebound or guarding; bowel sounds are decreased. His hemoglobin concentration is 14.2 g/dL, leukocyte count is 12,000/mm3, and platelet count is 280,000/mm3. Abdominal ultrasonography shows a dilated noncompressible appendix with distinct wall layers and echogenic periappendiceal fat. Intravenous fluid resuscitation is begun. Which of the following is the most appropriate next step in management? (Oral amoxicillin & Clavulanic acid VS Laparoscopic appendectomy)

Crohns -A) Signs/Symptoms ----1) Chronic diarrhea ----2) Abd pain ----3) Fever ----4) Enterovaginal fistula -------(passing gas/feces from the vagina) PID - Would also present w/ malodorous discharge from vagina - However, KEY SYMPTOMS are missing including ---1) EXTREMELY tender uterus → pt WOULD HAVE guarding & rebound tenderness -------[pt has NO rebound/guarding] ---2) Dyspareunia & abnormal uterine bleeding ---3) PID DOES NOT PRESENT w/ ------a) Diarrhea, abd pain

A 29F, presents d/t a 2-month Hx of abd pain, diarrhea, & low-grade fever. The pt has 2/3 non-bloody BMs daily. Her abd pain is crampy & intermittent & occasionally causes her to wake up from sleep. For the past week, she has been passing gas & malodorous discharge from the vagina. The pt has lost 8.8-lbs over the past few months. She was previously healthy. The pt works as a flight attendant & travels internationally. She has had a new BF for 3-months, & her only Med is OCP. The pt does not smoke, drink, or use drugs. Temp is 100.9F, BP is 110/70, pulse is 90/min, & RR is 14/min. PE shows lower abd tenderness w/out guarding/rebound. Which of the following is the most likely Dx? (Crohns VS PID)

Colonoscopy - Pt shows signs concerning for Crohns disease ***Key to differentiating Crohns from UC is CT findings*** -A) CT findings ----1) Most frequently involves the terminal ileum ----2) Air in rectum -B) Signs/Symptoms ----1) RLQ tenderness ----2) Diarrhea → w/ severe abd pain + cramps ----3) Anemia ----4) Wt loss -C) MGMT ----a) Dx tests -------1) Endoscopy + biopsy → DIAGNOSTIC TEST ---------a) This includes ------------aa) ± Colonoscopy w/ ileoscopy ------------bb) ± Capsule endoscopy --------2) CT/MRI ----------a) to identify active inflammation & complicaitons Upper GI series w/ a water-soluble contrast & small bowel follow-through - Would be used to investigate suspected structural lesions involving stomach/small bowel (STRICTURES) → most often indicated in suspected: ----b) Duodenal atresia ----c) Malrotation w/ volvulus ----d) Crohns disease

A 32F, presents to the ED 24-hours after the onset of severe abd cramps & nausea. During this period, she has vomited yellow-green material once & has had only 2 BMs, which were small & liquid. During the past 6-months, she has had less severe abd cramps assocaited w/ diarrhea and a 15-lbs wt loss. She says the cramps have become more severe during the past month. She has no Hx of serious illness & takes no meds. On arrival, she is in moderate distress. She appears pale. She is 5-ft, 5-in tall & weighs 114-lbs; BMI is 19. Temp is 99F, pulse is 90/min, BP is 166/78. Abd exam shows moderate distention w/ mild RLQ tenderness. Leuko Ct is 14K. A CT scan of the abd shows thickening fo the terminal ileum & air in the rectum. The colon is not dilated. Which of the following is the most appropriate next step in Dx? (Upper GI series w/ a water-soluble contrast & small bowel follow-through VS Colonoscopy)

Trail of PPI - Pt has signs concerning for GERD -A) Signs/Symptoms ----1) Retrosternal burning sensation ----2) Nocturnal cough & sore throat -B) MGMT ---a) Dx tests ------1) Trial of PPI (BEST INITAL TEST) ------2) Esophageal pH monitoring for 24 hours ---------a) (DIAGNOSTIC TEST) ------3) Barium swallow ---------a) Used if suspected strictures are the cause ------4) Barium swallow OR EGD ----------b) To RULE OUT HIATAL HERNIA ------5) Esophageal manometry ---------b) PRE-OPERATIVE ONLY ------6) EGD w/ biopsy ----------a) To rule out Barret's ----b) TMT -------1) Lifestyle mods -----------a) Eliminate caffeine, alcohol, smoking -----------b) Elevate head of bed -----------c) Eat dinner several hours before bed -----------d) Wt loss -------2) Meds -----------a) Antacids (FIRST LINE) -----------b) PPI/H2 blockers -------3) Nissen fundoplication -----------a) For refractory to Meds

A 32M, presents d/t a 3-month Hx of episodes of burning substernal chest pain & a sour taste in his mouth. He sometimes awakens at night w/ an acidic liquid in his mouth. He has had a non-productive cough for 2 months. He has smoked one pack of ciggs daily for 14 years. He drinks approximately six beers every week. He is 5-ft, 7-in tall & weighs 198-lbs; BMI is 31. Exam shows no other abnormalities. Which of the following is the most appropriate next step in MGMT? (Trial of PPI VS Upper GI endoscopy)

Gastric hemorrhage - Most common complication of PUD are: ---1) Gastric hemorrhage ---2) Gastic perforation -A) Signs/Symptoms ----1) Pain that presents AFTER meals -------a) Pain is NOT constant Gastric perforation - Also presents w/ epigastric pain after meals - However, key difference is ---1) PAIN PRESENTATION ------a) Perforation presents w/ CONSTANT pain that continues to worsen ---------[pt still has pain AFTER MEALS at this time, suggesting hemorrhage > perforation as the Dx]

A 35-year-old man comes to the physician because of a 2-month history of upper abdominal pain that occurs immediately after eating. The pain is sharp, localized to the epigastrium, and does not radiate. He reports that he has been eating less frequently to avoid the pain and has had a 4-kg (8.8-lb) weight loss during this time. He has smoked a pack of cigarettes daily for 20 years and drinks 3 beers daily. His vital signs are within normal limits. He is 165 cm (5 ft 5 in) tall and weighs 76.6 kg (169 lb); BMI is 28 kg/m2. Physical examination shows mild upper abdominal tenderness with no guarding or rebound. Bowel sounds are normal. Laboratory studies are within the reference range. This patient is at greatest risk for which of the following conditions? (Gastric perforation OR Gastric hemorrhage)

Abd-XR - Pt most likely presents w/ Toxic megacolon -A) RF ----1) Hx of UC & NON-COMPLIANT w/ meds -B) MGMT: -----a) Dx Tests ---------1) Abdominal XR -----------a) in ALL pts w/ toxic megacolon) -----b) TMT ---------1) Complete bowel rest ---------2) Correction of laboratory abnormalities ---------3) Broad-spectrum antibiotics -------------(e.g. ampicillin-gentamicin-metronidazole) ---------4) IV corticosteroids ---------5) Bowel decompression (via nasogastric or long intestinal tube) ---------6) Ex-LAP → indications include: ------------a) Features of bowel perforation/ischemia (abd guarding, rebound tenderness, air under diaphragm) ------------b) Peritoneal signs ------------c) Massive GI hemorrhage ------------d) No improvement after 24-72 hours ----------------[pt does not have these signs]

A 36-year-old man comes to the emergency department for the evaluation of recurrent bloody diarrhea for 4 weeks. During this time, he has also had intermittent abdominal pain. His symptoms have worsened over the past 2 days and he has also had fever and several episodes of nonbloody vomiting. He was diagnosed with ulcerative colitis three years ago but has had difficulty complying with his drug regimen. His temperature is 38.8°C (100.9°F), pulse is 112/min and regular, and blood pressure is 90/50 mm Hg. Abdominal examination shows a distended abdomen with no guarding or rebound; bowel sounds are hypoactive. Hemoglobin concentration is 10.1 g/dL, leukocyte count is 15,000/mm3, and erythrocyte sedimentation rate is 50 mm/h. Fluid resuscitation is initiated. In addition to complete bowel rest, which of the following is the most appropriate next step in the management of this patient? (Ex-LAP OR Abd-XR)

Inflammation & edema - Pt presents w/ Gastric outlet obstruction (GOO) - The pt also present w/ a duodenal ulcer: ---1) sever episodic pain that gets better w/ eating ------(pt has had weight gain) ---2) Pain that worsens at night & several hours after eating -A) Etiology of GOO includes: ----1) PUD -------a) Especially in pts w/ Hx of PUD → acute/chronic ----------aa) In ACUTE PUD → Inflammation & edema would be seen ----------bb) In CHRONIC PUD → scarring & fibrosis would be expected ---------------[Pt has presented w/ PUD symptoms for the LAST WEEK, which makes it ACUTE] ----2) GI malignancy (neoplastic growth) --------a) pt has no signs/Rf's for GI malignancy) ----3) Crohns ----4) Strictures ----5) Gastric bezoars

A 36-year-old man is brought to the emergency department because of multiple episodes of nonbilious emesis for 3 days. The vomitus consists of undigested food and he has also had dark brown emesis twice today. He has been having early satiety during this period. He has had progressive severe episodic epigastric pain and dyspepsia for the past week. The pain is partially relieved with food or over-the-counter antacids, but worsens at night and several hours after eating. He has also had a weight gain of 2 kg (4.4 lbs) during this period. His father underwent surgery for colon cancer 3 years ago. He has smoked one pack of cigarettes daily for the last 15 years. He drinks two to three beers daily. His temperature is 37.1°C (98.8°F), pulse is 106/min and blood pressure is 108/68 mm Hg. Examination shows dry mucous membranes. The abdomen is mildly tender to palpation in the left upper quadrant; there is a tympanitic mass in the epigastrium. A succussion splash is heard with a stethoscope when the patient is rocked back and forth at the hips. Bowel sounds are reduced. Rectal examination is unremarkable. Test of the stool for occult blood is positive. The remainder of the examination shows no abnormalities. Serum studies show: - Na+----------------------------------135 - K+------------------------------------3.3 - Cl------------------------------------97 - BUN----------------------------------46 - Glucose-----------------------------77 - Creatinine----------------------------1.4 Which of the following is the most likely cause of the pts vomiting? (Neoplastic growth OR Inflammation & edema OR Scarring & fibrosis)

Perforation - Presents w/: --1) Acute severe abd pain --2) Peritonitis -----Guarding & rebound tenderness --3) Signs of septic shock -----Hemodynamic instability -----Tachycardia -----Fever --4) RF's ----Hx of NSAID use Acute mesenteric ischemia - Can also present w/ acute, severe abd pain & signs of sepsis & absent bowel sounds - However, mesenteric ischemia usually presents w/: ---Loose bloody stools -----(pt does not have) ---Usually presents in pts > 60 ---Setting of hyper-coaguable states (shock, CV surgery (aortic aneurysm repair), or vasculitis ----(pt has none of these RFs)

A 36-year-old woman is brought to the emergency department after the sudden onset of severe, generalized abdominal pain. The pain is constant and she describes it as 9 out of 10 in intensity. She has hypertension, hyperlipidemia, and chronic lower back pain. Menses occur at regular 28-day intervals with moderate flow and last 4 days. Her last menstrual period was 2 weeks ago. She is sexually active with one male partner and uses condoms inconsistently. She has smoked one pack of cigarettes daily for 15 years and drinks 2-3 beers on the weekends. Current medications include ranitidine, hydrochlorothiazide, atorvastatin, and ibuprofen. The patient appears ill and does not want to move. Her temperature is 38.4°C (101.1°F), pulse is 125/min, respirations are 30/min, and blood pressure is 85/40 mm Hg. Examination shows a distended, tympanic abdomen with diffuse tenderness, guarding, and rebound; bowel sounds are absent. Her leukocyte count is 14,000/mm3 and hematocrit is 32%. Which of the following is the most likely cause of this patient's pain? (Acute mesenteric ischemia OR Perforation)

Proctocolectomy - Pt most likely presents w/ colorectal carcinoma - Dx findings for Colon cancer include: -A) MGMT → (suspected colorectal carcinoma) ----1) Colon Contrast enema ------a) Often in sigmoid colon ------b) Often infiltrative & partially obstructive & produces an apple-core lesion Antibiotic & bowel rest, TPN - This is the TMT for Sigmoid volvulus - Also presents w/ abd distention - However, key differences include: ----1) Usually seen in elderly pts --------[pt is 36] ----2) Coffee bean appearance in XR --------[pt uses CONTRAST ENEMA → a volvulus would not be seen with this] ----3) Pts Hx of UC points more to colorectal cancer → UC is NOT A RF for sigmoid volvulus

A 36M, presents b/c of a 4-day Hx of bloody diarrhea. He has a 16-yrs Hx of UC treated w/ corticosteroids. Temp is 99.5F, pulse is 110/min, RR is 16/min, & BP is 124/78. The abd is soft & mildly distended; there is mild, diffuse tenderness but no rigidity or guarding. Colon contrast show a constricting, apple-core, non-obstructing sigmoid mass. Which of the following is the most appropriate next step in MGMT? (Proctocolectomy VS Antibiotic therapy & bowel rest VS TPN)

Perform cutaneous drainage - Pt most likely presents w/ a liver abscess - A) Signs/Symptoms -----1) RUQ pain -----2) fever -B) Associated lab findings -----1) ↑ LFT's -----2) Leukocytosis -C) MGMT ----a) Dx tests -------1) XR -----------a) Hypo-echoic, heterogenous lesion w/ surrounding hyperemia Discontinue oral contraceptive - OCPs are associated w/ a hepatic adenoma - Also can present w/ RUQ paint - However, key differences include: ---1) USUALLY ASYMPTOMATIC -------a) Usually found incidentally (no fever, no ↑ LFTs, no leukocytosis) ----2) US/XR findings -------a) well-demarcated, heterogeneous lesion, w/ variable echogenicity -----------[US DOES NOT SHOW a well-demarcated lesion, shows a hypo-echoic lesion w/ hyperemia]

A 39-year-old woman comes to the physician because of fever, generalized fatigue, and chills for 1 week. She has been having dull right-side abdominal pain during this period. She has nausea. She has no history of travel outside the United States. She has type 1 diabetes mellitus. Current medications include insulin and an oral contraceptive. She appears ill. Her temperature is 40°C (104°F), pulse is 118/min, and blood pressure is 106/70 mm Hg. On examination, the liver is palpated 2-3 cm below the right costal margin and is tender. There are decreased breath sounds over the right lung base. The remainder of the examination shows no abnormalities. Laboratory studies show: - Hb--------------------------------------------14.1 - Leukocytes-----------------------------------17,1K - Segmented Neutros-------------------------74% - Eosinophils------------------------------------1% - Lymphocytes---------------------------------20% - Monocytes------------------------------------5% - Glucose---------------------------------------79 - Creatinine-------------------------------------1.1 - AST--------------------------------------------69 - ALT--------------------------------------------100 - ALP-------------------------------------------180 - Bilirubin (total)--------------------------------0.9 A right upper quadrant ultrasound is shown. Which of the following is the most appropriate next step in management? (Discontinue oral contraceptive OR Perform cutaneous drainage)

Pancreatic enzymes - Pt present w/ pancreatitis insufficiency in the setting of chronic pancreatitis and possibly pancreatic cancer -A) MGMT includes ----a) TMT → (Chronic Pancreatitis) -------1) Pancreatic enzymes (FIRST LINE TMT) -------2) Endoscopic stent placement (via ERCP) ------------a) pts who have pain resistant to pain therapy) ------------b) pts w/ pancreatic duct dilation >6mm) ----------------[Pt has none of these] -------3) Celiac ganglion block -----------a) Can offer temporary relief, but only AFTER other pain meds fail

A 42-year-old man comes to the physician because of severe epigastric pain for a week. The pain is constant and he describes it as 6 out of 10 in intensity. The pain radiates to his back and is worse after meals. He has had several episodes of nausea and vomiting during this period. He has taken ibuprofen for multiple similar episodes of pain during the past 6 months. He also has had a 5.4-kg (12-lb) weight loss over the past 4 months. He has a 12-year history of drinking 3 to 4 pints of rum daily. He has been hospitalized three times for severe abdominal pain in the past 3 years. He appears ill. His temperature is 37°C (98.6°F), pulse is 87/min, and blood pressure is 110/70 mm Hg. There is severe epigastric tenderness to palpation. Bowel sounds are normal. Cardiopulmonary examination shows no abnormalities. Laboratory studies show: - Hb--------------------------------------13.5 - Leukocyte Ct---------------------------7,8K - Glucose--------------------------------106 - Creatinine------------------------------1.1 - Amylase---------------------------------105 A CT of the abdomen is shown. Which of the following is the most appropriate long-term management for this patient? (Celiac ganglion block OR Pancreatic enzymes)

Dual-energy XR absorptiometry - Pt presents w/ PBC -A) Associated w/ other autoimmune disorders -----a) i.e., Sjogrens -A) Signs/Symptoms ----1) Pruritus ----2) Fatigue ----3) Anorexia ----4) Painful hepatomegaly -B) Signs/Symptoms → (indicating Liver failure) ----5) Xanthelasma, jaundice, Steatorrhea, light colored stool, Inflammatory arthropathy, Osteomalacia/osteoporosis, Portal HTN, varices, splenomegaly, Coagulopathy, Encephalopathy, Hypoalbuminemia -A) MGMT of PBC includes: -----a) Dx Tests ---------AA) Dx requires at least 2 of the following 3: -------------1) Increased alkaline phosphatase -------------2) Antimitochondrial antibodies -------------3) Non-suppurative destructive cholangitis and interlobular bile duct destruction --------BB) NON-REQUIRED TESTS -------------1) ) DEXA (Dual-energy XR absorptiometry) --------------a) b/c osteoporosis often accompanies PBC Extracorporeal shock wave lithotripsy - Although the pt presents w/ gallstones, she has no signs of cholelithiasis (colicky RUQ pain, fever, nausea, vomiting) - If she did have these signs, then this answer would be correct Trial of azathioprine & prednisone - Can be considered in autoimmune hepatitis - However, Key difference is ---1) BIOPSY FINDINGS -------a) Autoimmune hepatitis presents w/ periportal inflammation & with plasma cell infiltrates ----[pt has destruction of small- and medium-sized intrahepatic ducts → suggesting PBC]

A 42-year-old woman comes to the physician because of a 2-month history of generalized itching and worsening fatigue. There is no personal or family history of serious illness. She takes eye drops for dry eyes. She occasionally takes acetaminophen for recurrent headaches. She drinks one alcoholic beverage daily. Vital signs are within normal limits. Examination shows jaundice and a nontender abdomen. The liver is palpated 3 cm below the right costal margin and the spleen is palpated 2 cm below the left costal margin. Laboratory studies show: - Hb---------------------------------------------15.3 - Leukos-----------------------------------------8,4K - PT time----------------------------------------13 sec - Bilirubin total----------------------------------3.5 - Direct-------------------------------------------2.4 - AST----------------------------------------------79 - ALT----------------------------------------------73 - ALP----------------------------------------------396 A liver biopsy specimen shows inflammation and destruction of small- and medium-sized intrahepatic bile ducts. Magnetic resonance cholangiopancreatography (MRCP) shows multiple small stones within the gallbladder and a normal appearance of extrahepatic bile ducts. Which of the following is the most appropriate next step in management? (Trial of azathioprine & prednisone OR Dual-energy XR absorptiometry OR Extracorporeal shock wave lithotripsy)

Hct - The best prognostic indicators for acute pancreatitis ON ADMISSION are: --1) Leukocytosis (WBC > 16K) --2) AST >250 --3) Glucose >200 --4) Age >55 --5) LDH >350 AFTER 48 HOURS --1) Ca+ <8 --2) Hct drop > 10% --3) Arterial pO2 <60 --4) BUN increase >5 --5) Base deficit >4 --6) Fluid needs >6L Of the choices, only Hct is an answer choice Lipase - Lipase (& amylase) is a DIAGNOSTIC lab test (does not test for severity) AST/ALT ratio - Only AST is a diagnostic indicator (not the ratio)

A 43-year-old man is brought to the emergency department because of severe epigastric pain and vomiting for 6 hours. The pain radiates to his back and he describes it as 9 out of 10 in intensity. He has had 3-4 episodes of vomiting during this period. He admits to consuming over 13 alcoholic beverages the previous night. There is no personal or family history of serious illness and he takes no medications. He is 177 cm (5 ft 10 in) tall and weighs 55 kg (121 lb); BMI is 17.6 kg/m2. He appears uncomfortable. His temperature is 37.5°C (99.5°F), pulse is 97/min, and blood pressure is 128/78 mm Hg. Abdominal examination shows severe epigastric tenderness to palpation. Bowel sounds are hypoactive. The remainder of the physical examination shows no abnormalities. Laboratory studies show: - Hb--------------------------------------------13.5 - Hct-------------------------------------------62% - Leukocytes-----------------------------------13,8K - Na+-------------------------------------------134 - Cl----------------------------------------------98 - Glucose---------------------------------------180 - Creatinine-------------------------------------0.9 - Amylase---------------------------------------150 - Lipase----------------------------------------347 (N 14-280) - LDH------------------------------------------360 - AST--------------------------------------------19 - ALT--------------------------------------------18 - ALP--------------------------------------------66 - Bilirubin (total)--------------------------------0.8 Which of the following laboratory studies is the best prognostic indicator for this patient's condition? (Lipase OR Hct OR AST/ALT ratio)

Antibiotic therapy - Pt present w/ signs concerning for Cholecystitis -A) RF ----1) Fatty food ingestion w/in hours of onset of pain -B) Signs/Symptoms -----1) RUQ/epigastric pain --------a) Pain is Constant, severe, prolonged (>4-6 hours) & worsens --------b) Can radiate to back/shoulder -----2) Fever, -----3) Nausea, vomiting, anorexia -C) PE findings -----1) Inspiratory arrest w/ deep palpation of the right upper sub-costal area -D) MGMT -----a) Dx Test --------1) Abd US (BEST INITAL) → associated findings: -----------a) Gallbladder wall thickening -----------b) Pericholecystic fluid -----------c) Sonographic Murphy sign --------2) Cholescintigraphy (HIDA) (GOLD STD DIAGNOSTIC) ------------a) FAILURE to visualize the gallbladder at 4-hours post-injection (N 30-60mins) or 30-mins post-morphine augmentation ------b) TMT ----------1) NPO + IV fluids ----------2) Antibiotics (piperacillin-Tazobactam) --------------a) in pts w/ fever & PMHx of DM ----------3) Laparoscopic cholecystectomy -------------a) w/in 48 hours (2-3 days) AFTER INITIATION of antibiotics ERCP - In pts w/ Stone present on US, ERCP would only be used if pt had a stone obstructing the CBD (dilation) -----[Pt has a NORMAL CBD] - Additionally, A stone would presents w/ INTERMITTENT and pain -----[pt has CONSTANT pain that is worsening]

A 44-year-old woman comes to the emergency department because of a 10-hour history of severe nausea and abdominal pain that began 30 minutes after eating dinner. The pain primarily is in her right upper quadrant and occasionally radiates to her back. She has a history of type 2 diabetes mellitus and hypercholesterolemia. Current medications include metformin and atorvastatin. She is 163 cm (5 ft 4 in) tall and weighs 91 kg (200 lb); BMI is 34 kg/m2. Her temperature is 38.8°C (101.8°F), pulse is 100/min, respirations are 14/min, and blood pressure is 150/76 mm Hg. Abdominal examination shows right upper quadrant tenderness with guarding. A bedside ultrasound shows a gall bladder wall measuring 6 mm, pericholecystic fluid, sloughing of the intraluminal membrane, and a 2 x 2-cm stone at the neck of the gallbladder. The common bile duct appears unremarkable. Laboratory studies show leukocytosis and normal liver function tests. Intravenous fluids are started, and she is given ketorolac for pain control. Which of the following is the most appropriate next step in management? (ERCP OR Antibiotic therapy OR Elective laparoscopic cholecystectomy in 6 weeks)

Cholecystoenteric fistula - Presentation includes: --1) Colicky, intermittent abd pain → (its associated w/ gallstones) --2) Vomiting --3) Complications ----a) SBO → Gallstones are associated w/ SBO → SBO presentation includes: -------1) High pitched bowel sounds -------2) Distended & tympanitic abdomen w/ diffuse tenderness -------3) Collapsed rectum --4) Fever --5) Labs -----1) Leukocytosis Viscus perforation - Pt would have a PMHx sig for ulcers - Pain that is associated w/ ulcers is typically CONSTANT ----(pt has INTERMITTENT/COLICKY pain)

A 46-year-old woman comes to the emergency department because of intermittent abdominal pain and vomiting for 2 days. The abdominal pain is colicky and diffuse. The patient's last bowel movement was 3 days ago. She has had multiple episodes of upper abdominal pain that radiates to her scapulae and vomiting over the past 3 months; her symptoms subsided after taking ibuprofen. She has coronary artery disease, type 2 diabetes mellitus, gastroesophageal reflux disease, and osteoarthritis of both knees. Current medications include aspirin, atorvastatin, rabeprazole, insulin, and ibuprofen. She appears uncomfortable. Her temperature is 39°C (102.2°F), pulse is 111/min, and blood pressure is 108/68 mm Hg. Examination shows dry mucous membranes. The abdomen is distended and tympanitic with diffuse tenderness; bowel sounds are high-pitched. Rectal examination shows a collapsed rectum. Her hemoglobin concentration is 13.8 g/dL, leukocyte count is 14,400/mm3, and platelet count is 312,000/mm3. Which of the following is the most likely cause of this patient's findings? (Viscus perforation OR Cholecystoenteric fistula)

cholecystoenteric fistula - Pt presents w/ SBO - SBO is most likely d/t gallstone ileus - Supporting signs include: --1) Presenting pain described as colicky/intermittent & diffuse -----(typical of gallstones) --2) XR findings: ----1) Show gas in the biliary tree (pneumobilia) Common causes of SBO include: --1) Post-op adhesions (most common) --2) Neoplasms --3) Hernias --4) Appendicitis/diverticulitis/intra-abdominal abscess --5) Gallstone --6) Intussusception/volvulus --7) Congenital stricture (pyloric stenosis) --8) Crohns Bowel infarction - This describes mesenteric ischemia (not SBO)

A 46-year-old woman comes to the emergency department because of intermittent abdominal pain and vomiting for 2 days. The abdominal pain is colicky and diffuse. The patient's last bowel movement was 3 days ago. She has had multiple episodes of upper abdominal pain that radiates to her scapulae and vomiting over the past 3 months; her symptoms subsided after taking ibuprofen. She has coronary artery disease, type 2 diabetes mellitus, gastroesophageal reflux disease, and osteoarthritis of both knees. Current medications include aspirin, atorvastatin, rabeprazole, insulin, and ibuprofen. She appears uncomfortable. Her temperature is 39°C (102.2°F), pulse is 111/min, and blood pressure is 108/68 mm Hg. Examination shows dry mucous membranes. The abdomen is distended and tympanitic with diffuse tenderness; bowel sounds are high-pitched. Rectal examination shows a collapsed rectum. Her hemoglobin concentration is 13.8 g/dL, leukocyte count is 14,400/mm3, and platelet count is 312,000/mm3. An x-ray of the abdomen is shown. Which of the following is the most likely cause of this patient's findings? (Bowel infarction OR cholecystoenteric fistula)

Postive anti-smooth muscle antibodies - Pt most likely presents w/ autoimmune hepatitis - Presents w/ similar findings to PBC, including: ---1) Same biopsy results ---2) Associated w/ autoimmune disease - However, pt lacks other symptoms that would support PBC, such as:Cholestatic presentation (jaundice, pruritus) Postive anti-mitochondrial antibodies - Associated w/ PBC - Symptoms supporting PBC differential ---1) Same biopsy results ---2) Associated w/ autoimmune disease - KEYS TO DIFFERENTIATING AUTOIMMUNE HEPATITIS FROM PBC ARE: ----1) No Cholestatic presentation (no jaundice, no pruritus)

A 50-year-old woman comes to the physician for the evaluation of fatigue over the past 6 months. During this period, the patient has also had a 5 kg (11-lb) weight loss. She has a history of Hashimoto thyroiditis. She is sexually active with her husband only. She does not smoke. She drinks one glass of wine per day. She does not use illicit drugs. Her only medication is levothyroxine. Temperature is 37°C (98.6°F), pulse is 70/min, and blood pressure is 110/70 mm Hg. Abdominal examination shows tenderness in the right upper quadrant with no rebound or guarding. Laboratory studies show a serum alanine aminotransferase level of 190 U/L, serum aspartate aminotransferase level of 250 U/L, and serum total bilirubin level of 0.6 mg/dL. Liver biopsy shows plasma cell infiltration and areas of periportal piecemeal necrosis. Further evaluation of this patient is most likely to show which of the following findings? (Postive anti-mitochondrial antibodies OR Postive anti-smooth muscle antibodies)

Anal fissure - Pt has signs & RF concerning for anal fissure, including: -A) RF ----1) Chronic constipation -B) Signs/symptoms ----1) Small amount of bright blood on tissue ----2) Relief w/ sitz bath ----3) Pain w/ or after defecation ----4) POSTERIOR location b/c that area is poorly perfused -C) MGMT ----a) TMT ------1) Stool softeners ------2) Sitz baths ------3) topical analgesics ------4) Topical vasodilators Thrombosed external hemorrhoid - Also presents as acute, severe anal pain - however, key difference is ---1) THROMBOSIS present → pt would have a tender & swollen, bulging, blue-purple colored nodule ------[pt DOES NOT have a nodule]

A 50M w/ chronic constipation has had anal pain after each bowel MVMT for 2 weeks. He has also noted a small amount of bright blood on the toilet tissue. Pain is relieved by sitz baths between BMs. Which of the following is the most likely cause of these symptoms? (Thrombosed external hemorrhoid VS Anal fissure)

Omeprazole - Pt presents w/ signs for GERD -A) MGMT ----1) Omeprazole → (FIRST-LINE) -B) Signs/symptoms ----1) Chest pain/epigastric pain often in association w/ a large meal or trigger food -------(i.e., meat) -C) MGMT ----a) Dx tests -------1) Upper GI endoscopy ----------a) Mucosal irritation & inflammation → which can presents as erythema & multiple linear erosions (reflux esophagitis) Aluminum hydroxide-alginic acid (Gaviscon) - This is an antacid - Pt has heartburn UNRELIEVED by antacids

A 52M presents 3-weeks after an episode of dysphagia while eating meat. He has had heartburn for 2 years; it has been unrelieved by antacids for 9 months. He has smoked two packs of ciggs daily for 35 years. Exam shows no abnormalities. HCT is 36%. A PBS shows microcytic, hypochromic erythrocytes. Upper GI endoscopy shows erythema of the mucosa between a hiatal hernia & the mid- esophagus w/ multiple linear erosions. Which of the following is the most appropriate pharmacotherapy? (Omeprazole VS Aluminum hydroxide-alginic acid (Gaviscon)

CT scan of abdomen w/ contrast - Pt most likely has diverticulitis -A) Signs/Symptoms -----1) LLQ MASS & lower abdominal pain --------a) A painful rectal MASS can be palpable on DRE in 20% of pts) --------b) Hypoactive bowel sounds) -----2) Leukocytosis & fever -----3) Altered bowel habits --------a) Constipation/diarrhea) --------b) Urinary frequency, urgency, dysuria ------5) Vomiting -B) PE findings -----1) Hypoactive bowel sounds -C) MGMT ----a) Dx Tests --------1) CT scan of abd & pelvis w/ oral & IV contrast (DIAGNOSTIC TEST)→ findings include: -----------a) Bowel wall thickening (>4cm) -----------b) Inflammation of peri-colonic fat w/ fat stranding -----------c) Presence of colonic diverticula --------2) Colonoscopy -----------a) Should be performed ≥6 weeks​ after the resolution of acute diverticulitis​ to rule out an underlying colorectal malignancy → unless a colonoscopy has been performed within the previous year **SIDE NOTE → Colonoscopy & barium enema are CONTRAINDICATED in setting of ACUTE diverticulitis as they can perforate the inflamed bowel wall ----b) TMT ---------1) Clear liquid diet/NPO x2-3 days ---------2) Antibiotics for 7-14 days -------------a) Metronidazole, PLUS -------------b) Ciprofloxacin OR levofloxacin OR Trimethoprim-sulfamethoxazole --------------c) amoxicillin-clavulanate or moxifloxacin -----------------cc) Alternatives --------3) Pain control --------4) IV fluids as necessary​ --------5) Surgery → indicated in pts w/: -------------a) No response to medical treatment -------------b) Frequent recurrent infections -------------c) Complications -----------------cc) perforation, fistula formation, abscess, stricture, obstruction ---------6) Emergent colectomy with end colostomy (ie, Hartmann's procedure -------------a) Diverticulitis complicated by colonic perforation and diffuse peritonitis Abdominal US - Only if CT scan is not available

A 55-year-old woman comes to the emergency department because of a 24-hour history of severe lower abdominal pain. She has had two episodes of nonbloody vomiting today and has been unable to keep down food or fluids. She has not had a bowel movement since the day before. She has hypertension, hyperlipidemia, and osteoarthritis. She had a cholecystectomy 5 years ago. She has smoked one pack of cigarettes daily for the last 20 years. Current medications include chlorthalidone, atorvastatin, and naproxen. Her temperature is 38.8°C (101.8°F), pulse is 102/min, respirations are 20/min, and blood pressure is 118/78 mm Hg. She is 1.68 m (5 ft 6 in) tall and weighs 94.3 kg (207.9 lbs); BMI is 33.4 kg/m2. Abdominal examination shows a soft abdomen with hypoactive bowel sounds. There is moderate left lower quadrant tenderness. A tender mass is palpable on digital rectal examination. There is no guarding or rebound tenderness. Laboratory studies show: - Leukocytes-------------------------------17K - Hb-----------------------------------------13.3 - HCT---------------------------------------40% - PLTS---------------------------------------188K - Na+----------------------------------------138 - K+------------------------------------------4.1 - Cl------------------------------------------101 - HCO3--------------------------------------22 - BUN----------------------------------------18.1 - Creatinine----------------------------------1.1 Which of the following is the most appropriate to confirm the Dx? (Abdominal US OR CT scan of abdomen w/ contrast)

CT abdomen - Pt presents w/ chronic pancreatitis & has an INCREASED risk for pancreatic adenocarcinoma -A) MGMT -----a) Dx test ---------1) CT of abdomen (DIAGNOSTIC) -------------a) Best initial test as it can also rule out cancer -----b) TMT ---------1) Whipples (pancreaticduodenoectomy) ---------2) ERCP ---------3) Biliary stenting -------------a) Specific for jaundice ---------4) Ablation of celiac nerves -------------a) Specific for back pain Abdominal US - Not sensitive, d/t overlying bowel gas & does a poor job of identifying necrosis - Can be used to detect late findings of chronic pancreatitis, such as pancreatic ductal calcifications, and/or ductal irregularity → both of which are pathognomonic for pancreatitis

A 57-year-old man comes to the physician because of a 3-week history of abdominal bloating and increased frequency of stools. He describes the stools as bulky, foul-smelling, and difficult to flush. He also has a 4-month history of recurrent dull upper abdominal pain that usually lasts for a few days, worsens after meals, and is not relieved with antacids. He has had a 10-kg (22-lb) weight loss in the past 4 months. He has no personal or family history of serious illness. He has smoked 1 pack of cigarettes daily for 37 years. He has a 12-year history of drinking 6 to 8 beers daily. He is 160 cm (5 ft 3 in) tall and weighs 52 kg (115 lb); BMI is 20 kg/m2. His vital signs are within normal limits. Abdominal examination shows mild epigastric tenderness without rebound or guarding. Bowel sounds are normal. The remainder of the examination shows no abnormalities. Which of the following is the most appropriate next step in diagnosis? (CT of abdomen OR Abdominal US)

VIPoma -A) Signs/Symptoms ----1) SEVERE, PROFUSE WATERY diarrhea -------a) Diarrhea Despite not eating ----2) Facial flushing ----3) Abd pain ----4) Muscle cramps ----5) Wt loss/anorexia -B) Complications ----1) Dehydration ----2) AKI ----3) Heart arrhythmias -B) Assocaited labs ----1) Hypokalemia -C) MGMT ----a) Dx tests -------1) Helical-contrast CT ----b) TMT -------1) Replacement of fluids & electrolyte correction ------2) Somatostatin analogues ---------a) Octreotide/lanreotide ------3) Glucocorticoids ---------a) For refractory to somatostatin analogues Gastrinoma - Also presents w/ diarrhea, abd pain, - However, key differences include: ---1) DIARRHEA is NOT PROFUSE & WATERY ---2) OTHER KEY SYMPTOMS -------a) Recurrent ulcers (jejunal & duodenal) -------b) ± hematemesis, melena, hematochezia Somatostatinoma - Also presents as a neuroendocrine tumor w/ wt loss, - However, key differences include: ---1) PRESENTATION → Includes ------a) DM ------b) Steatorrhea ------c) Achlorhydria ---------(inhibition of gastrin)

A 57M, is admitted d/t a 1-month Hx of profuse, watery diarrhea w/ facial flushing. During this period, he has had fatigue & a 10-lbs wt loss. The diarrhea has not resolved w/ use of OTC antidiarrheal meds. He has no sig PMHx & takes no other meds. Pulse is 110/min, & BP is 100/70. Exam shows no abnormalities. IV fluid resuscitation is begun. His stool output is 5L over the next 24-hours; his stool K+ conc is increased. Serum studies show a pH of 7.56 & a K+ conc of 2.8. A CT of the abd shows a pancreatic mass. Which of the following is the most likely Dx? (VIPoma VS Somatostatinoma VS Gastrinoma)

Nutritional replacement - Pt is experiencing re-feeding syndrome → presentation includes: -A) Signs & symptoms: ---1) Hypophosphatemia ---2) Hypokalemia, ---3) Weakness, ---4) Arrhythmia ---5) CHF ---6) Rhabdomyolysis ---7) Neuro dysfunction → (seizures, paresthesia) -All of which develop AFTER re-introduction of nutrition in a setting of chronic malnourishment → (evidenced by anorexia, malignancy) Hypermetabolic syndorme - Typically occurs in the setting of: ---1) Severe burns ---2) Sepsis ---3) Hyperthyroidism Hepatic metabolic impairment - This is associated w/ massive tumor burden - Presentation includes: ---1) Loss of synthetic function → (coagulopathy, hypoalbuminemia) ---2) Ammonia accumulation → (asterixis, hepatic encephalopathy) -However, electrolyte abnormalities & arrhythmia are not characteristic Paraneoplastic syndrome - Typically presents w/ different electrolyte abnormalities, sucks as: ---1) Hyponatremia → (SIADH) ---2) Hypercalcemia → Lung cancer

A 58M, previously healthy male, is evaluated d/t a 2-mnths Hx of early satiety, postprandial epigastric pain & vomiting, & unintended wt loss. PE shows cachexia & a palpable epigastric mass. Upper GI endoscopy reveals a large pre-pyloric mass causing gastric outlet obstruction; biopsy of the mass is constant w/ poorly differentiated adenocarcinoma. Abd CT reveals thickening of the gastric wall w/ adjacent fat infiltration, enlarged LNs, & multiple hepatic metastatic lesions. The disease is considered un-resectable, & the pt undergoes laparoscopic gastrojejunostomy for palliation of gastric outlet obstruction. Post-op, he tolerates tube feeding but experiences worsening weakness. The pt also has frequent episodes of non-sustained ventricular tachycardia. Labs show: -------------At admission----------------Post-op day 2 - Hb------------9.9----------------------------9.8 - Leukos-------8,8K---------------------------10K - Na+-----------130----------------------------135 - K+-------------3.7-----------------------------2.8 - Cr-------------1.1-----------------------------0.8 - Glucose------90-----------------------------120 - Phosphorus--3.1-----------------------------1.2 Which of the following is the most likely cause of this pts worsening condition? (Hepatic metabolic impairment OR Hypermetabolic syndrome OR Nutritional replacement OR Paraneoplastic syndrome)

Upper endoscopy - Pt presents w/ signs concerning for Gastric outlet obstruction (GOO) - GOO is most often caused by Malignancy (gastric cancer) - MGMT is based on looking for the malignancy -A) MGMT ----a) Dx tests -------1) Abd CT/Upper endoscopy → ENDOSCOPY PREFERRED ----------a) BOTH can be used as Dx test in GOO ----------b) Endoscopy → PREFERRED b/c -------------1) Can take a biopsy at same time -------------2) Can also provide short-term TMT w/ stent placement/dilation -----b) TMT for (GI CANCER) --------1) Resection w/ LN dissection + Chemo & radiation -----------a) EXCEPT in case of early disease --------2) Palliative radiation -----------a) For non-resectable cases Abd US - Can be used for screening in gastric cancer - However, not used as a Dx test in gastric cancer

A 63-year-old man comes to the emergency department because of a 2-day history of persistent vomiting after meals. The vomit consists of undigested food and a clear fluid. His last attempt to eat was 6 hours ago and his last bowel movement was 3 days ago. For the past 3 weeks, the patient has had worsening nausea and early satiety. He also reports a 6.8-kg (15-lb) weight loss over the past 4 months. He has hypercholesterolemia treated with atorvastatin. The patient drinks 2-3 glasses of wine daily. He appears thin. His temperature is 37.2°C (99.0°F), pulse is 80/min and blood pressure is 120/65 mm Hg. Examination shows dry oral mucosa and conjunctival pallor. Abdominal examination shows epigastric fullness with mild tenderness. A sloshing sound is heard upon auscultation of the epigastrium when rocking the patient back and forth. Which of the following is most likely to confirm the diagnosis? (Abd US VS Upper Endoscopy)

Discontinue amiodarone - Pt is showing ELEVATED LIVER values -A) AE of amiodarone include: -----1) Hepatotoxicity, -----2) Thyroid dysfunction, -----3) Pulmonary fibrosis, -----4) Cardiac arrhythmias, -----5) Photosensitivity, -----6) Corneal deposits, -----7) Peripheral neuropathy Discontinue acetaminophen - Although it can cause elevated liver values, they are usually ONLY seen w/ acetaminophen OD - Additionally, the liver values would be EXTREMELY ELEVATED (in the thousands)

A 63-year-old man comes to the physician for a routine health maintenance examination. He feels well. He has a history of hypertension, atrial fibrillation, bipolar disorder, and osteoarthritis of the knees. Current medications include lisinopril, amiodarone, lamotrigine, and acetaminophen. He started amiodarone 6 months ago and switched from lithium to lamotrigine 4 months ago. The patient does not smoke. He drinks 1-4 beers per week. He does not use illicit drugs. Vital signs are within normal limits. Examination shows no abnormalities. Laboratory studies show: - Na+-------------------------------137 - K+---------------------------------4.2 - Cl---------------------------------105 - HCO3-----------------------------24 - BUN--------------------------------14 - Cr----------------------------------0.9 - ALP---------------------------------82 - AST--------------------------------110 - ALT---------------------------------115 Which of the following is the most appropriate next step in MGMT? Discontinue acetaminophen VS Discontinue amiodarone)

AV malformation in the colonic wall - Pt most likely has angiodysplasia -A) Etiology ----1) Most common cause of GI bleeding in pts >60 ----2) ESRD → 30% of patients w/ ESRD have angiodysplasia -B) Signs/Symptoms ----1) Anemia symptoms -------a) fatigue -------b) Palpitations -------c) SOB ----2) Bloody stools Tissue ischemia d/t decreased splanchnic perfusion - Describes mesenteric Ischemia - Also Presents w/ GI bleeding in Pt w/ a PMHx sig for ESRD & DM - However, key differences include: ---1) PRESENTATION SYMPTOMS ------a) Mesenteric ischemia presents w/ periumbilical pain, diarrhea, high fever -------[pt has NONE of these symptoms] ---2) PRESENTATION TIME-LINE ------a) Mesenteric ischemia Presents w/ Sudden onset of symptoms ---------[pt has a GRADUAL onset; 6 months of worsening symptoms]

A 65-year-old man comes to the physician because of progressively worsening fatigue for 6 months. During this time, he has also had shortness of breath and palpitations on exertion. He has noticed blood in his stools on three separate occasions in the past 4 months. He has type 2 diabetes mellitus and end-stage renal disease. He drinks two to four beers daily. He does not smoke. His pulse is 95/min and blood pressure is 120/70 mm Hg. Examination shows pale conjunctivae. The abdomen is soft with no organomegaly. Rectal examination is unremarkable. His hemoglobin concentration is 7.2 g/dL, hematocrit is 32%, and mean corpuscular volume is 68 μm3. Which of the following is the most likely underlying cause of this patient's bleeding? (AV malformation in the colonic wall OR Tissue ischemia d/t decreased splanchnic perfusion)

Gastric cancer - Presentation includes: -A) RFs ---1) Asian -B) Signs & symptoms ---1) Wt loss ---2) Persistent mid-epigastric Abd pain, ---3) Nausea ---4) Gastric outlet obstruction → (causing occasional emesis) ---5) Hepatomegaly → d/t METs -C) Labs ---1) Fe+ deficiency anemia MGMT -1) Esophagogastroduodenoscopy -2) Abd CT → for staging Chronic viral hepatitis - Can also present w/ nausea, wt loss, and mild transaminitis - However, DOES NOT typical present w/: ---1) Mild, mid-epigastric pain ---2) Emesis ---3) Microcytic anemia

A 65M, presents d/t 4 months of abd pain. nausea, & occasional emesis. The pain is deep, mid-epigastric, & non-radiating. There is no consistent relationship between the pain & food intake. He has no difficulty swallowing, hematemesis, black stools, blood in the stool, or diarrhea. The pt thinks he may have lost wt recently. he emigrated to the US from China 20 years ago but frequently travels to China to visit family. He has no sig PMHs. The pt does not smoke, drink, or use drugs. Temp is 98.4F, BP is 110/78, pulse is 78/min, RR are 16/min. BMI is 20. Heart and lung sounds are normal. The upper abd is mildly tender to palpation but non-distended. Hepatomegaly is present. Bowel sounds are normoactive. Labs show: - Hb---------------------------------7.8 - MCV-------------------------------72 - PLTS-------------------------------380K - Leukos-----------------------------8,1K ---neutros---------------------------60% ---Eosinophils-----------------------1% ---Lymphos-------------------------29% - Total bili---------------------------0.6 - Albumin----------------------------2.8 - ALP--------------------------------182 - AST---------------------------------74 - ALT---------------------------------64 - Lipase------------------------------55 (N 5-70) -Lab testing last year was normal Which of the following is the most likely cause of this pts symptoms? (Gastric cancer OR Chronic viral hepatitis)

Cardiac stress test - The pt presents w/ signs of atypical angina - Presentation includes: --1) Pain that is exacerbated by eating large meals & exercise --2) RF's ----a) Diabetes ----b) Smoking Hx ----c) HTN ----d) Obesity ----e) PAD --3) Absent foot pulses ----(suggests decreased blood flow) --4) MGMT -----a) Dx Tests --------1) Cardiac stress test ECG→ via either ------------a) Physical → Treadmill + ECG ------------b) Pharmacological + ECG ----------------1) Dobutamine CT angio of the abdomen - Used in suspected mesenteric ischemia - Also presents w/ abd discomfort & nausea following meals - However, key differences include: ---1) MISSING KEY SYMPTOMS OF ISCHEMIA, including: -------a) abdominal bloating -------b) Food aversion -------c) Wt loss ---2) CHARACTERIZATION OF PAIN --------a) Pain in mesenteric ischemia DOES NOT WORSEN w/ activity ------------[pt describes pain that worsens w/ meals AND ACTIVITY]

A 68-year-old man comes to the physician because of recurrent episodes of nausea and abdominal discomfort for the past 4 months. The discomfort is located in the upper abdomen and sometimes occurs after eating, especially after a big meal. He has tried to go for a walk after dinner to help with digestion, but his complaints have only increased. For the past 3 weeks he has also had symptoms while climbing the stairs to his apartment. He has type 2 diabetes mellitus, hypertension, and stage 2 peripheral arterial disease. He has smoked one pack of cigarettes daily for the past 45 years. He drinks one to two beers daily and occasionally more on weekends. His current medications include metformin, enalapril, and aspirin. He is 168 cm (5 ft 6 in) tall and weighs 126 kg (278 lb); BMI is 45 kg/m2. His temperature is 36.4°C (97.5°F), pulse is 78/min, and blood pressure is 148/86 mm Hg. On physical examination, the abdomen is soft and nontender with no organomegaly. Foot pulses are absent bilaterally. An ECG shows no abnormalities. Which of the following is the most appropriate next step in diagnosis? (CT angio of the abdomen OR Cardiac stress test)

Open emergency surgery - Pt presents w/ AAA -A) Signs/Symptoms ---1) Severe abd pain ± radiates to left flank ---2) Pulsatile mass -B) MGMT → BASED ON HEMODYNAMIC STABILITY ---AA) Hemodynamically STABLE PTs -------a) Dx test ----------1) CT of abd w/ contrast -------b) TMT ----------1) β-blockers (esmolol/labetalol) ----------2) Non-dihydropyridine calcium channel blockers (ie, diltiazem, verapamil) --------------a) Alternative to β-blockers ----------2) ± Vasodilators (nitroprusside/nicardipine) ---BB) Hemodynamically UN-stable -------a) Dx Test -----------1) Transesophageal Echo (TEE) -------b) TMT ------------1) Emergent surgery

A 69-year-old man is brought to the emergency department because of severe abdominal pain radiating to his left flank for 30 minutes. He is weak and has been unable to stand since the onset of the pain. He vomited twice on the way to the hospital. He has not passed stools for 3 days. He has hypertension, coronary heart disease, and peptic ulcer disease. He has smoked half a pack of cigarettes daily for 46 years. Current medications include enalapril, metoprolol, aspirin, simvastatin, and pantoprazole. He appears ill. His temperature is 37°C (98.6°F), pulse is 131/min, respirations are 31/min, and blood pressure is 82/56 mm Hg. Pulse oximetry on room air shows an oxygen saturation of 92%. The lungs are clear to auscultation. Cardiac examination shows no abnormalities. Examination shows a painful pulsatile abdominal mass. Intravenous fluid resuscitation is begun. Which of the following is the most appropriate next step in management? (CT scan of abd w/ contrast OR Open emergency surgery)

CT angio of abdomen - Pt presents w/ mesenteric ischemia (CHRONIC) -A) Signs/Symptoms → Differ between ACUTE VS CHRONIC ischemia -aa) Chronic ischemia ------1) Postprandial pain (epigastric or periumbilical) ------2) Fear of eating (sitophobia) ------3) Weight loss ------4) History of vascular disease ------5) Nausea, vomiting, diarrhea -bb) Acute ischemia ------1) Constant, dull, diffuse abd pain ------2) Bloody diarrhea ------3) Nausea, vomiting -B) RF's -----1) CAD -----2) CABG -----3) Smoking Hx -----4) Age (>60), F>M -C) PE findings (Chronic) -----1) Pain disproportionate to physical exam findings -----2) Abdominal bruit -----3) Decreased peripheral pulses​ -D) MGMT ----a) Dx tests -------1) CT Angio (GOLD STANDARD) -------2) Abd XR/MRA ----b) TMT -------1) Open/endovascular revascularization -------2) Warfarin RUQ US - Test for Cholelithiasis → which presents similarly to mesenteric ischemia - Also presents w/ postprandial pain not relieved w/ PPI, Bloating and/or diarrhea - However, key differences include: ----1) CHOLELITHIASIS DOES NOT PRESENT W/ --------a) No food aversion --------b) Wt loss ----2) PAIN CHARACTERIZATION --------a) Pain in cholelithiasis typically INTERMITTENT -----------[Although the word EPISODIC is used, the pts pain is described as WORSENING episodic pain which indicates pain progression to CONSTANT]

A 71-year-old man comes to the physician accompanied by his wife because of a 6-month history of worsening episodic upper abdominal pain and an 8-kg (17.6-lb) weight loss. The pain is dull, nonradiating, worse after eating, and occasionally associated with bloating and diarrhea. His symptoms have not improved despite 4 weeks of treatment with omeprazole. He attributes his weight loss to recently eating very small portions and avoiding fatty foods. He has hypertension and hypercholesterolemia. He underwent a coronary artery bypass graft operation for coronary artery disease 8 years ago. Current medications include lisinopril, metoprolol, atorvastatin, and aspirin. He has smoked a pack of cigarettes daily for 20 years and drinks 1-2 beers daily. His pulse is 79/min and blood pressure is 138/89 mm Hg. Examination shows a soft abdomen without tenderness to palpation or guarding. Which of the following is most likely to confirm the diagnosis? (RUQ US OR CT angio of abdomen)

Pancreatic adenocarcinoma - Pt presents w/ classic signs, including: ---1) Painless jaundice ------(NON-TENDER palpation on PE) ---2) Conjugated hyperbilirubinemia ------(Pale stools & dark urine) ---3) Pruritus ---4) Anorexia ---5) Unexplained Wt loss ---6) ABD US findings: ------a) Non-tender gallbladder ---------(Anechoic sub-hepatic CYSTIC mass) ------b) Dilation of intrahepatic & extra hepatic bile ducts ----7) RF's ------a) Smoking Hx ------b) Advanced age Acute cholangitis - Typically presents w/ RUQ pain, fever, jaundice - However, pts would also have: tachycardia, upper abd guarding & tenderness ----[pts abd is soft & non-tender] ---1) Acute Cholangitis DOES NOT present w/a soft, cystic mass & unexplained wt loss

A 72-year-old man is brought to the physician by his son because of gradually progressive yellow discoloration of his skin and generalized pruritus for the past 2 weeks. During this period, his appetite has decreased and he has had a 6.3-kg (14-lb) weight loss. He reports that his stool appears pale and his urine is very dark. Three years ago, he had an episode of acute upper abdominal pain that was treated with IV fluids, NSAIDs, and dietary modification. He has stopped drinking alcohol since then; he used to drink 1-2 beers daily for 40 years. He has smoked a pack of cigarettes daily for the past 50 years. His vital signs are within normal limits. Physical examination shows yellowing of the conjunctivae and skin. The abdomen is soft and nontender; a soft, cystic mass is palpated in the right upper quadrant. Serum studies show: - AST----------------------------------------------32 - ALT----------------------------------------------34 - ALP---------------------------------------------192 - Bilirubin total------------------------------------5.6 - Direct--------------------------------------------4.8 Abdominal ultrasonography shows an anechoic cystic mass in the subhepatic region and dilation of the intrahepatic and extrahepatic bile ducts. Which of the following is the most likely diagnosis? (Acute cholangitis OR Pancreatic adenocarcinoma)

ERCP - Pt presents w/ signs & Hx concerning for cholangiocarcinoma -A) RF ----1) Alcohol -B) Signs/Symptoms ----1) RUQ pain ----2) Wt loss -C) Associated labs → (KEY TO Dx) ----1) ↑ ALP ----2) SLIGHTLY increased AST -D) MGMT -----a) Dx tests --------1) Abd US -----------a) Dilated intrahepatic ducts -----------b) SMALL GALLBLADDER --------------a) Specific to cholangiocarcinoma --------2) ERCP ------------a) Used to evaluate extent & location of tumor -----b) TMT ---------1) Stent ---------2) Chemotherapy Liver biopsy - Would be used is HCC was suspected - HCC also presents w/ RUQ pain, decreased appetite & wt loss in the setting if chronic alcohol use - However, Key Differences include: ---1) US FINDINGS → would show Inflammatory changes (necrosis, fibrosis) ------[Pt has DILATED INTRAHEPATIC DUCTS & SMALL GALLBLADDER] ---2) LABS → Would show ↑ AST & ALT ------[pt has SLIGHTLY increased AST] ---3) OTHER COMMON SYMPTOMS, are missing including: ------a) Ascites, palmar erythema, spider angiomata ------b) Gynecomastia, asterixis, caput medusae

A 72M, presents b/c of a 4-month Hx of right-sided upper abd pain; he has decreased appetite resulting in a 20-lbs Wt loss during this period. He has consumed 4-oz whiskey daily for 50 years. He is 6-ft, 4-in tall & weighs 180-lbs. BMI is 22. Temp is 98.6F, BP is 140/80. Abd exam shows no abnormalities. Labs show: - Hct--------------------------------38% - MCV-------------------------------90 - Bilirubin total----------------------2 --- Direct----------------------------1.4 - ALP--------------------------------500 - AST--------------------------------50 - LDH-------------------------------200 - Lipase------------------------------42 (N 14-280) Abd US shows dilated intrahepatic ducts in both lobes & a small gallbladder. A CT of the Abd shows no additional abnormalities. Which of the following is the next best step in MGMT? (ERCP VS Liver Biopsy)

Esophagogastroduodenoscopy - Pt presents w/signs & symptoms concerning for gastric cancer, including: -A) RFs ---1) Asian -B) Signs/symptoms ---1) Wt loss ---2) Epigastric tenderness & fullness ---3) Pain that is worse when eating & ONLY SOMETIMES relieved w/ antacids -MGMT ---1) Esophagogastroduodenoscopy H/pylori testing - Pt has signs similar to PUD - however, he has symptoms concerning for gastric cancer which must take priority - Symptoms are: ---1) Wt loss ---2) Pain that is worse after eating & is only sometimes received w/ antacids ---3) Asian decent ---4) NO PREVIOUS Hx of ulcers

A 72M, presents w/ upper abd pain & wt loss. The abd pain is worse when eating food & is sometimes relieved w/ OTC antacids. He has not had dysphagia, melena, or rectal bleeding. The pt has a Hx of HTN which is well-controlled w/ chlorthalidone. He does not take other prescription/OTC meds & does not smoke, drink, or use drugs. The pt emigrated from South Korea 20 years ago. Temp is 98.2F, BP is 110/70, pulse is 84/min, & RR are 14/min. BMI is 17.2. Mucus membranes are dry. Cardiopulmonary exam shows no abnormalities. Epigastric fullness & tenderness are present, but there is no hepatosplenomegaly. Peripheral pulses are full & capillary refill is normal. Which of the following is the best next step in MGMT? (Esophagogastroduodenoscopy OR H.pylori testing)

Sigmoid volvulus -A) Signs/symptoms ----1) Constipation ----2) Abd bloating ----3) Distention ----4) Nausea & vomiting -B) MGMT ----a) Dx tests -------1) Abd XR ---------aa) Coffee-bean ----b) TMT -------1) Sigmoidoscopy w/ placement of a rectal tube ----------a) Detorsion & to facilitate stool passage & maintain decompression ------2) Surgery ----------a) Colopexy/colectomy ----------b) For recurrent sigmoid volvulus Mechanical bowel obstruction -Also presents w/ nausea & vomiting, Obstipation, cramping and Abd pain & a RF for SBO is previous surgery - However, key differences include: ---1) Abd XR findings → would show: ------a) Air in bowel DIFFUSELY ------b) Abd free air (under diaphragm) -----c) Reduced bowel wall enhancement ---------[pt has characteristic COFFEE-BEAN sign] ---2) PE findings → Typically presents w/: ------a) High-pitched tinkling sounds ------b) Hypoactive/absent bowl sounds ------c) Peritoneal signs ------d) Atypical abd pain → pain that CONTINUALLY WORSENS ----------[None of these findings are on pt's PE] ----3) OTHER CHARACTERISTIC SIGNS -------a) Fever -------b) Tachycardia -------c) Leukocytosis (>10K) -----------[pt has NONE of these signs]

A 77F, is admitted b/c of severe cramping pain in the lower abd. The pain has been constant for the past 16 hours. The abd is markedly distended. A McBurney incisional scar is noted in the RLQ. Abd XR is attached. Which of the following is the most likely Dx? Sigmoid volvulus VS Mechanical obstruction of small intestine)

Entamoeba -1) Presents w/: ---a) Diarrhea >2 wks → (sign it is typically PARASITIC) ---b) Starts off watery then progresses to bloody ---c) Colitis symptoms → (abd pain , bloody diarrhea) -----(abd can be located in different quadrants) Vibrio - Causes profuse WATERY diarrhea ---[pt has BLOODY diarrhea]

A 7M, presents to the ED d/t bloody diarrhea. The pt was previously in good health. He began having mild watery stools 18 days ago that have gradually become more frequent. For the past 3 days, the stools have become grossly bloody. The family just returned from a 4-wks trip to Pakistan to visit extended family. Temp is 98,2F; other vitals are normal. PE shows mild left lower quadrant tenderness & active bowel sounds. There is no rebound tenderness. Stool samples are sent for micro studies. Which of the following is the most likely cause of this pts symptoms? (Vibrio OR Entamoeba)

Biliary cyst -A) Etiology: -----1) Most common in asians & children <10 -B) Signs/Symptoms - CLASSIC TRIAD -----1) Jaundice -----2) Pain -----3) Palpable mass -----4) Anorexia/vomiting -----5) Darkening of urine & light colored stools -----6) Itching -C) MGMT ----a) Dx tests -------1) US ----------a) Present as saccular/fusiform DILATIONS of the biliary tree ----------b) Dilated gallbladder but is NORMAL -------2) ERCP ----b) TMT --------1) Excision of cyst ------------a) Relives obstruction & decreases risk of malignancy Biliary atresia - Would also present w/ cholestasis (Acholic stools, darkening urine, & jaundice) - However, key differences include: --1) PRESENTATION -----a) Biliary atresia Develops w/in first few weeks (4-8 weeks) of life ----------[pt is 2 yrs old; he is TOO OLD to have atresia] --2) PE FINDINGS ------a) Atresia DOES NOT PRESENT W/ palpable abd mass --3) US findings: -----1) ATRESIA of the extra hepatic ducts -----2) Absent CBD small/absent gallbladder -----3) Absent/irregular gallbladder W/OUT BILIARY DILATION ---------[pt has NORMAL gallbladder & BILIARY Dilation] -----4) Triangular cord sign ---------a) increased echogenicity along the anterior wall of the portal vein above the porta hepatis -MGMT ---a) Dx tests ------1) Intraoperative cholangiogram (GOLD STD DIAGNOSTIC TEST) ------2) US ------3) Hepatobiliary scintigraphy ------4) Percutaneous liver biopsy -----------a) to differentiate suspected biliary atresia from other etiologies of intrahepatic cholestasis ---b) TMT ------1) Hepatoportoenterostomy (Kasai procedure) ---------a) should be performed in all infants at the time of biliary atresia diagnosis and ideally before 60 days of age ------2) Ursodeoxycholic acid ------3) Liver transplantation Mirizzi syndrome - Also presents w/ Cholestasis & dilation of the hepatic duct & gallbladder - However, most often seen in pts w/ history significant for chronic hemolytic anemia (Hereditary spherocytosis)

A previously healthy 2-year-old girl is brought to the physician because of a 1-week history of yellow discoloration of her skin, loss of appetite, and 3 episodes of vomiting. Her parents also report darkening of her urine and light stools. During the last 2 days, the girl has been scratching her abdomen and arms and has been crying excessively. She was born at 38 weeks' gestation after an uncomplicated pregnancy and delivery. Her family emigrated from Japan 8 years ago. Immunizations are up-to-date. Her vital signs are within normal limits. Examination shows jaundice of her skin and sclerae. Abdominal examination shows a mass in the right upper abdomen. Serum studies show: - Bilirubin (total)-----------------------------5 - Direct---------------------------------------4.2 - AST------------------------------------------20 - ALT------------------------------------------40 - GGT------------------------------------------110 Abdominal ultrasonography shows dilation of the gall bladder and a fusiform dilation of the extrahepatic bile duct. Which of the following is the most likely diagnosis? (Biliary atresia OR Biliary cyst OR Mirizzi syndrome)

Hypersensitivity to gliadin - Urinary D-xylose should normally increase after consumption → this pts D-xylose stays the same, which indicates a problem w/ sugar absorption - This describes Celiacs disease Autoimmune mediated inflammation of colon - This describes UC - Also presents w/ recurrent abd pain, diarrhea anemia, wt loss - However, pts typically have blood in stool - D- Xylose test would be normal

A previously healthy 20-year-old woman comes to the physician because of recurrent abdominal cramps, bloating, and diarrhea for 4 months. She describes her stools as greasy, foul-smelling, and difficult to flush. During this time she has had a 6-kg (13.2-lb) weight loss. She has no personal or family history of serious illness. Physical examination shows pallor and cheilitis. Laboratory studies show a hemoglobin concentration of 11 g/dL. Serum concentrations of electrolytes, urea nitrogen, and creatinine are within the reference range. Test of the stool for occult blood is negative and stool microscopy reveals no pathogens and no leukocytes. Analysis of a 24-hour stool sample shows 12 g of fat. The patient is asked to consume 25 g of d-xylose. Five hours later, its concentration is measured in urine at 2 g (N = > 4 g/5 h). The test is repeated after a two-week course of rifaximin, but the urinary concentration of d-xylose remains the same. Which of the following is the most likely diagnosis? (Hypersensitivity to gliadin OR Autoimmune mediated inflammation of colon)

Acute cholecystitis -A) PE findings → KEY TO DIFFERENTIATION ----1) Tenderness to palpation in the RUQ --------a) + Guarding --------b) NO REBOUND -B) Signs/symptoms ----1) Fever, ----2) Biliary Colic (RUQ/epigastric pain) -------a) Constant, severe, prolonged (4-6 hours) ----3) Anorexia, nausea, vomiting -C) RF ----1) Pregnancy ----2) Gallstones ----3) TPN -C) Associated labs -----1) Leukocytosis -D) MGMT -----a) Dx tests --------1) Abd US -----------a) Gallbladder wall thickening/dilated gallbladder -----------b) Pericholecystic fluid/sub-serosal edema (double wall sign) -----------c) Sonographic McMurphy sign → inspiratory arrest during gallbladder palpation w/ US transducer Acute appendicitis - Also presents w/ Abd pain, fever, nausea & vomiting, & leukocytosis - However, key differences include: ---1) PAIN PRESENATION ------a) Typically presents w/ intermittent, colicky abd pain ---------[pt has CONSTANT abd pain] ------b) Pain begins in the epigastrium but then shifts to the RLQ 12-18 hours after initial onset ---------[pt has CONSTANT RUQ pain] ---2) PE findings -----a) pt would present w/ McBurneys sign, Rovsings sign, and Psoas sign --------[pt only has VOLUNTARY GUARDING]

A previously healthy 27F, at 28-weeks is admitted to the hospital 2 days after the onset of constant right upper abd pain. Her symptoms have worsened over the past 12 hours. During this period, she also has had decreased appetite, nausea & vomiting, & decreased fetal mvmts. She has had no contractions, vaginal bleeding, abnormal vaginal discharge, or change in bowel mvmts. Temp is 100.4F, Pulse is 95/min, BP is 138/70. the lungs are clear to auscultation. Heart sounds are normal. Exam shows a soft abd & tenderness in the RUQ w/ voluntary guarding & no rebound. Bowel sounds are normal. Fundal Ht is 28-cm. Labs show: - Hb--------------------------------12 - Leukos----------------------------14K - PLTS------------------------------140K - PT time---------------------------12sec (INR = 1.06) - Na+--------------------------------142 - K+----------------------------------4.5 - Cl----------------------------------104 - Total bili---------------------------1.8 - ALP--------------------------------268 - AST--------------------------------40 - ALT--------------------------------44 -----URINE----- - Glucose---------------------------neg - Protein---------------------------trace - Ketones----------------------------2+ Which if the following is the most likely DX? (Acute appendicitis VS Acute cholecystitis)

RUQ US - Pt presents w/ signs concerning for acute pancreatitis -A) MGMT: ----a) Dx Tests -------1) Serum Amylase & lipase levels -------2) Imaging GENERALLY NOT REQUIRED, However it is indicated in: -------------1) The pancreatitis is severe -------------2) Tumor is suspected -------------3) There is suspicion for duct obstruction ----------2.1) CT ± contrast ----------2.2) US -------------a) To look for gallstones, biliary sludge, or blockage → which are a MAJOR causes of acute pancreatitis ----------2.3) ERCP -------------a) USED AS LAST/FINAL test & used TO CONFIRM Dx & provide therapeutic (if stone is the cause) ----b) TMT -------1) NPO, -------2) IV fluids, -------3) Analgesics (fentanyl) ERCP - Only indicated in acute pancreatitis w/ evidence of: ---1) Choledocholilithiasis -------(dilated CBD +/- stone, ↑ bilirubin, ↑ ALP, ↑ GGT) ---2) Acute cholangitis -------(fever, jaundice, Murphy sign, leukocytosis, ↑ bilirubin, ↑ ALP, ↑ GGT)

A previously healthy 31-year-old woman comes to the emergency department because of sudden, severe epigastric pain and vomiting for the past 4 hours. She reports that the pain radiates to the back and began when she was having dinner and drinks at a local brewpub. Her temperature is 37.9°C (100.2°F), pulse is 98/min, respirations are 19/min, and blood pressure is 110/60 mm Hg. Abdominal examination shows epigastric tenderness and guarding but no rebound. Bowel sounds are decreased. Laboratory studies show: - Hct---------------------------------------------43% - Leukos-----------------------------------------9K - Na+---------------------------------------------140 - K+-----------------------------------------------4.5 - Ca+----------------------------------------------9 - Lipase-------------------------------------------170 (N <50) - Amylase-----------------------------------------152 - ALT-----------------------------------------------140 Intravenous fluid resuscitation is begun. Which of the following is the most appropriate next step in management? (ERCP OR RUQ US)

Urea breath test - Pt present w/ signs supporting a Dx of PUD > GERD -A) Signs/Symptoms: ----1) Intermittent burning epigastric pain -------a) Pain is usually related to food -----------aa) Gastric ulcer → Pain increased w/ food -----------bb) Duodenal ulcer → Pain decreased w/ food ----2) Usually w/meals --------a) Gastric ulcer → Pain increased w/ food --------b) Duodenal ulcer → Pain decreased w/ food -B) PE findings: ------a) Mild epigastric pain on palpation w/out guarding or rebounding -C) MGMT ----a) Dx Tests --------1) Endoscopy (DIAGNOSTIC) --------2) Biopsy ------------a) To exclude malignancy --------3) Urea breath test and/or stool antigen ------------a) Must investigate H.pylori b/c it is the most common cause of PUD ----b) TMT --------1) H.pylori eradication therapy -----------after confirmation w/ urea breath test PPI - This is the TMT for GERD - GERD also presents w/ epigastric pain that usually occurs after meals and is common in male patients >45 - However, key differences include: ---1) PAIN PRESENTATION for GERD -------a) Pain that is worse at night & described as a RETROSTERNAL BURNING SENSATION -----------[pt has INTERMITTENT EPIGASTRIC PAIN that worsens w/ meals] ---2) MISSING KEY SYMPTOMS OF GERD -------a) Regurgitation of food -------b) Wt loss -------c) Aspiraton -------d) Odynophagia -------e) GI bleeding/unexplained anemia ----------[pt has NONE of these KEY symptoms]

A previously healthy 37-year-old man comes to the physician for the evaluation of a 8-week history of intermittent burning epigastric pain. During this period, he has also felt bloated and uncomfortable after meals. He has not had weight loss or a change in bowel habits. He has no personal or family history of serious illness. He takes no medications. He does not smoke. He drinks 1-3 beers per week. Vital signs are within normal limits. Abdominal examination shows mild epigastric tenderness on palpation without guarding or rebound tenderness. Bowel sounds are normal. The remainder of the examination shows no abnormalities. Which of the following is the most appropriate next step in management? (PPI OR Urea breath test OR H. pylori eradication therapy

Re-feeding syndrome - Pt presents w/ abnormal labs, RFs, & signs/symptoms concerning for re-feeding syndrome -A) Assocaited labs ----1) Hypophosphatemia ----2) Hypomagnesemia -------a) Pt has a level of 1.6 → technically not HYPO but still low -B RFs ----1) Occurs w/ TPN → usually in malnourished pts or pts who have not had an appetite for 5-7 days Adynamic ileus - Also presents w/ abd pain & delayed passage of stool following surgery (usually 3-5 days post-op) & is assocaited w/ morphine - However, key differences include: ---1) KEY SYMPTOMS ------a) Nausea/vomiting ------b) Failure to tolerate oral diet ------c) Abdominal distention, bloating ----------[pt does not have] ------d) Diffuse, constant abdominal pain ----------[pt has MILD TENDERNESS] ------e) Delayed passage of stool AND FLATUS ----------[pt has flatus] ---2) ABNORMAL LABS include HYPOKALEMIA --------[pt has hypoMg+ & Hypophosphatemia]

A previously healthy 42F, is admitted to the hospital for TMT for acute biliary pancreatitis. On hospital day 5, she is passing flatus occasionally, but she has not had a BM for 2 days. She has a loss of appetite but does not have nausea. Current meds are a pt controlled morphine pump & IV omeprazole. Temp is 101.3F, pulse is 80/min, RR are 20/min, & BP is 130/75. Abd exam shows mild tenderness in the epigastrium & central abd to direct palpation; there is no rebound. A CVC is inserted, & TPN is begun. On hospital day 6, labs show: - Na+-------------------------------135 - K+---------------------------------4 - Cl---------------------------------102 - HCO3-----------------------------26 - Mg+--------------------------------1.6 (N 1.5-2) - Ca+--------------------------------8.6 - Phosphorus-----------------------2 (N 3-4.5) Which if the following is the most explanation for these lab findings? (Adynamic ileus VS Re-feeding syndrome)

Elevated CA19-9 - Pt most likely has pancreatic adenocarcinoma - Presentation includes cholestasis: ---(pale stools, jaundice, itching) - Abdominal US findings: ---1) Dilation the biliary & pancreatic duct + mass in head of pancreas Elevated anti-neutrophil cytoplasmic antibodies - This would be seen in Primary sclerosing cholangitis - This also presents w/ cholestasis (jaundice, itching, pale stools) - However, Abdominal US would show: --1) Beading of the intra & extra hepatic bile ducts -----(pt shows dilation of the biliary & pancreatic ducts → which is associated w/ pancreatic adenocarcinoma)

A previously healthy 48-year-old man comes to the physician because of a 3-week history of progressively worsening jaundice, generalized itching, and epigastric discomfort. He also complains of nausea and loss of appetite. His stools have looked like clay for the past week. He has returned from a vacation in Thailand one week ago, where he got a new tattoo. He is sexually active with multiple partners and does not use protection. His vital signs are within normal limits. Examination shows jaundice and scleral icterus. Superficial excoriations are seen on all limbs. Abdominal examination shows no abnormalities. Serum studies show a fasting glucose level of 198 mg/dL, total bilirubin concentration of 10.6 mg/dL, direct bilirubin concentration of 9.8 mg/dl, and alkaline phosphatase activity of 450 U/L. Abdominal ultrasonography shows dilation of the biliary and pancreatic ducts and a 3-cm hypoechoic solid mass with irregular margins in the head of the pancreas. An elevation of which of the following serum findings is most specific for this patient's condition? (Elevated anti-neutrophil cytoplasmic antibodies OR Elevated CA19-9)

Colonoscopy - Pt presents w/ melena -A) MGMT ----a) Dx tests -------1) Esophagogastroduodenoscopy (EGD) (PREFERRED INITIAL) -------2) Colonoscopy -----------a) Used when EGD is negative b/c can be caused by diverticulosis/colon cancer Capsule Endoscopy - Mainly used to visualize the small bowel - However, in cases of melena, Dx tests should be focused on the most common sites of bleeding, including: ---1) Esophagus/duodenum → --------a) Esophagogastroduodenoscopy ---2) Colon --------a) Colonoscopy ---3) Small bowel --------a) Capsule endoscopy

AMBOSS A 42-year-old man is brought to the emergency department because of several episodes of black, tarlike stool and lightheadedness over the past couple of days. He underwent an appendectomy at the age of 16. He has hypertension and tension headaches. He drinks 4 beers daily. Current medications include amlodipine and naproxen. His temperature is 36.7°C (98°F), pulse is 100/min, and his blood pressure is 100/70 mm Hg. The lungs are clear to auscultation bilaterally. The abdomen is soft, nondistended, and nontender. The liver is palpated 3 cm below the right costal margin. His hemoglobin concentration is 10 g/dL. Intravenous fluid resuscitation is begun. An esophagogastroduodenoscopy is performed, which does not show bleeding. Which of the following is the most appropriate next step in management? (Capsule endoscopy VS Colonoscopy)

Mesenteric angiography - Pt presents w/ signs concerning for mesenteric ischemia -A) Signs/Symptoms ----1) Peritoneal signs -------a) Pain out-of-proportion to Pe findings → Pts pain is so severe she has refused to eat, however, PE findings reveal MILD, DIFFUSE TENDERNESS -------b) Vomiting & nausea -------c) Hematochezia -B) MGMT ----a) Dx tests -------1) Mesenteric angiography → GOLD STANDARD -------2) Abd CT findings ----------aa) Focal/segmental bowel wall thickening Esophagoduodenostomy - Used in gastrectomy

An 82F, is admitted to the hospital b/c she has refused to eat b/c of a 2-day Hx of severe abd pain. She has DM II treated w/ insulin. Temp is 98.6F, pulse is 102/min, BP is 110/85. Abd exam shows mild, diffuse tenderness to palpation. Labs show: - Na+--------------------------------138 - K+----------------------------------4 - Cl----------------------------------103 - HCO3------------------------------20 - Glucose---------------------------202 An Abd CT shows thickening of the wall of the small bowel. Which of the following is the most appropriate next step in MGMT? (Mesenteric angiography VS Esophagoduodenostomy)

Stretching of the glisson capsule - Pt meets all the criteria for HELLP syndrome - HELLP syndrome DOES NOT REQUIRE HTN/proteinuria -A) Signs/symptoms -----1) Epigastric pain & nausea that is worse at night ---------a) Stretching of Glisson capsule from hepatic swelling is common -B) MGMT ----a) Dx Tests -------1) US ----------a) To investigate abd pain Inflammation of the lower esophageal mucosa - This describes GERD-induced esophagitis, which is common in pregnancy - Also presents w/ epigastric pain that is worse at night - However, key differences include ---1) GERD DOES NOT PRESENT w/ HELLP -------a) Hemolytic anemia -------b) Elevated liver enzymes -------c) Low PLTS

An otherwise healthy 28-year-old primigravid woman at 30 weeks' gestation comes to the physician with a 5-day history of epigastric pain and nausea that is worse at night. Two years ago, she was diagnosed with a peptic ulcer and was treated with a proton pump inhibitor and antibiotics. Medications include folic acid and a multivitamin. Her pulse is 90/min and blood pressure is 130/85 mm Hg. Pelvic examination shows a uterus consistent in size with a 30-week gestation. Laboratory studies show: - Hb---------------------------------------------8.6 - PLTS-------------------------------------------95K - Bilirubin total----------------------------------1.5 - LDH--------------------------------------------705 - AST---------------------------------------------80 URINE - pH----------------------------------------------6.2 - Protein------------------------------------------2+ - WBC--------------------------------------------neg - Bacteria------------------------------------occasional - Nitrites----------------------------------------neg Which of the following best explains this patient's symptoms? (Stretching of the glisson capsule OR Inflammation of the lower esophageal mucosa)

Hep C -A) MGMT ----a) Dx tests -------1) HCV antibody test (BEST INITIAL) -------2) PCR ----------a) to Confirm infection ----b) TMT -------1) Ledipasvir-sofosbuvir -------2) Sofosbuvir-velpatasvir -------3) Glecaprevir-pibrentasvir Mixed Cryoglobulinemia -A) MGMT ----a) Dx tests -------1) Diagnosed w/ Associated findings ----------a) Palpable purpura ----------b) Low complement levels ----------c) Circulating serum cryoglobulins ----------d) Elevated Rheumatoid factor -----b) TMT -------1) Ledipasvir-sofosbuvir -------2) Sofosbuvir-velpatasvir -------3) Glecaprevir-pibrentasvir

Compare/Contrast the Dx tests & MGMT for Hep C Vs Mixed cryoglobulinemia

Ischemic colitis - Presentation includes: --1) RF's -----a) AAA repair (can dislodge embolus) -----b) Major blood loss during surgery --2) Symptoms (presents in 3 stages) -----a) Stage 1 ---------Crampy abdominal pain w/ Bloody stools -----b) Stage 2 ---------Diffuse abd pain w/ bloating, absent bowel sounds, & cessation of stools -----c) Phase 3 ----------peritonitis & shock Post-op ileus - Also Presents w/ diffuse abdominal pain & distension, nausea/vomiting, absence of flatus, absent/reduction of bowel sounds - additionally, Surgery is a RF - However, key difference is ---1) BOWEL MVMTS ------a) Post-op ileus presents w/ ABSENT BM's post-surgery ---------[Pt has had 2 bloody stools Post-op]

Five days after undergoing an open abdominal aortic aneurysm repair, a 68-year-old woman has crampy abdominal pain. During this period, she has also had two episodes of loose, bloody stools. Her surgery was complicated by severe blood loss requiring the administration of vasopressors and multiple transfusions. Cefazolin was administered as a perioperative antibiotic. The patient has hypertension, hypercholesterolemia, and coronary artery disease. The patient has smoked 2 packs of cigarettes daily for 50 years and drinks 3-4 glasses of wine every week. Her current medications include hydrochlorothiazide, atorvastatin, amlodipine, aspirin, and metoprolol. She appears ill. Her temperature is 38.0°C (100.4°F), pulse is 110/min, and blood pressure is 96/58 mm Hg. Physical examination shows a distended abdomen with absent bowel sounds. The abdomen is exquisitely tender to palpation in all quadrants. The lungs are clear to auscultation. Cardiac examination shows an S4 gallop. An x-ray of the abdomen shows air-filled distended bowel. Which of the following is the most likely diagnosis? (Post-op ileus OR Ischemic colitis)

Retroperitoneal hemorrhage - Pt presents w/ paralytic ileus - Common causes of paralytic ileus include: --1) Retroperitoneal hemorrhage -----(in setting of MVC w/ bilateral bruising of flanks & absent psoas shadow) --2) Pain meds (morphine) ----(pt is taking Tylenol, which will not cause paralytic ileus)

Four days after having been admitted to the hospital for a pulmonary contusion and whiplash injury sustained in a motor vehicle collision, a 66-year-old woman complains of severe pain in her right flank and muscle spasms. She also has nausea with two episodes of vomiting and abdominal bloating. Her pain had previously been well controlled with acetaminophen every 6 hours. She underwent umbilical hernia repair surgery two years ago. She takes sertraline for depression. Her temperature is 36.5°C (97.7°F), pulse is 99/min, respirations are 17/min, and blood pressure is 102/72 mm Hg. After administration of 0.5 L of crystalloid fluids, blood pressure improves to 118/79 mm Hg. Multiple ecchymoses are present over the anterior abdominal wall in a pattern that follows the course of a seatbelt. There are ecchymoses of the flanks bilaterally. Bowel sounds are absent. There is tenderness to palpation in all four quadrants with voluntary guarding. Her hemoglobin is 7.9 g/dL, leukocyte count is 8,500/mm3, platelet count is 350,000/mm3, prothrombin time is 11 seconds, and activated partial thromboplastin time is 33 seconds. An x-ray of the abdomen shows obliteration of the right psoas shadow and uniform distribution of gas in the small bowel, colon, and rectum without air-fluid levels. Which of the following is the most likely explanation for this patient's symptoms? (Effect of pain meds OR Retroperitoneal hemorrhage)

ERCP - Pt presents w/ post-cholecystectomy syndrome → caused by a sphincter of ODDI dysfunction -A) Sign/Symptoms: ----1) biliary-type (intermittent/episodic) pain after cholecystectomy ----2) Elevated cholestatic pattern (LFT's) ----3) Dilated CBD w/out evidence of stones -B) MGMT ----a) Dx tests --------1) Abd US (BEST INITIAL TEST) --------2) Cholescintigraphy (HIDA scan) (DIAGNOSTIC) --------3) ERCP -----------a) Allows for visualization & TMT if a stone if found -----b) TMT --------1) NPO & IV fluids & pain control --------2) Antibiotics --------3) Laparoscopic cholecystectomy ------------a) W/in 48 hours AFTER INITAL antibiotics -------4) ERCP ----------a) Can be BOTH diagnostic & curative CT scan of abdomen - Would be used if suspected pancreatic adenocarcinoma - Can also Present w/abd pain that radiates to the back, abnormal LFT's, - However, key differences include ---1) MISSING KEY SYMPTOMS of pancreatic cancer, including: ------a) Wt loss, anorexia ------b) GOO (succussion splash) ------c) Painless jaundice ------d) Enlarged gallbladder ------e) Migratory thrombophlebitis ----------[pt has NONE of these KEY symptoms]

Fourteen days after a laparoscopic cholecystectomy for cholelithiasis, a 45-year-old woman comes to the emergency department because of persistent episodic epigastric pain for 3 days. The pain radiates to her back, occurs randomly throughout the day, and is associated with nausea and vomiting. Each episode lasts 30 minutes to one hour. Antacids do not improve her symptoms. She has hypertension and fibromyalgia. She has smoked 1-2 packs of cigarettes daily for the past 10 years and drinks 4 cans of beer every week. She takes lisinopril and pregabalin. She appears uncomfortable. Her temperature is 37°C (98.6° F), pulse is 84/min, respirations are 14/min, and blood pressure is 127/85 mm Hg. Abdominal examination shows tenderness to palpation in the upper quadrants without rebound or guarding. Bowel sounds are normal. The incisions are clean, dry, and intact. Serum studies show: - AST----------------------------------90 - ALT----------------------------------95 - ALP----------------------------------213 - Bilirubin (total)----------------------1.3 - Direct--------------------------------0.7 - Amylase-----------------------------52 (N 25-125) Abdominal ultrasonography shows dilation of the common bile duct and no gallstones. Which of the following is the most appropriate next step in management? (ERCP OR CT scan of abd)

Small bowel obstruction -Presentation includes: -A) Etiology ----a) Post-op adhesions ----b) Neoplasms ----c) Hernias ----d) Crohns ----e) Other -------1) Congenital stricture, Appendicitis, diverticulitis, intra-abdominal abscess, Intussusception, volvulus, congenital strictures, Traumatic intramural hematoma, gallstone -B) Signs & symptoms ----1) Nausea, vomiting, obstipation -------a) Pain exacerbated by eating & relieved by vomiting ----2) Atypical abd pain --------a) localized, continuing, worsening ----3) Fever, tachycardia -C) Labs ----1) WBC's > 10K -D) PE findings ----1) Peritoneal signs (rigidity, guarding, rebound tenderness) ----2) High pitched tinkling sounds (tympanitic) ----3) Hypoactive/absent bowel sounds Gastric outlet obstruction -Can present w/ vomiting of undigested food -However, key differences include ---1) PRESENTATION ------a) GOO DOES NOT present w/ bile in vomitus --------[pt had clear-green liquid initially (bile)] -----b) GOO DOES present w/ succussion splash --------[pt DOES NOT have succussion splash] ---2) OCCURS IN SPECIFIC SETTING ------a) GI CACNER ------a) Pyloric stenosis ------b) PUD ------c) Cancer ------d) Pancreatic pseudocyst ---------[pt has none of these]

NBME CK 7 A 27F, presents b/c of a 5-day Hx of headache, severe, diffuse abd pain, & nausea & vomiting. The pain is exacerbated by eating & relieved by vomiting. The vomitus initially consisted of a large amount of semi-solid material mixed w/ a clear green liquid but now consists of clear yellow liquid only. She has not had any other symptoms. She has Crohns treated w/ prednisone; she was instructed to taper her dosage 2 weeks ago. Additional meds include mesalamine & Azathioprine. Her last menstrual period was 7-wks ago. She is sexually active w. one male partner, & they use condoms inconsistently. She appears ill & is in moderate distress. Temp is 101.2F, pulse is 120/min, RR are 12/min, & BP is 90/50. Exam shows dry oral mucosa & pale conjunctivae. The abd is distended, diffusely tender, & tympanitic. Bowel sounds are decreased. Pelvic exam shows no abnormalities. Labs show: -Hct--------------------31% -Leuko Ct--------------15K -Amylase---------------300 (N 25-125) -Lipase-----------------9 (N 14-280) Which of the following is the most likely Dx? (Gastric outlet obstruction OR Small bowel obstruction)

Acute Cholecystitis -A) Signs/Symptoms ----1) RUQ pain -------a) Constant, severe, & prolonged (>4-6 hours) -------b) May radiate to right shoulder/back -------c) Usually preceded by Fatty food ingestion ----2) Fever ----3) Anorexia, nausea, vomiting -B) PE findings ----1) Murphys signs (SPECIFIC FINDING) -C) MGMT ----a) Dx tests -------1) Abd US (BEST INITAL TEST & DIAGNOSTIC) -----------a) Gallbladder wall thickening -----------b) Double-wall sign (Pericholecystic fluid/sub-serosal edema) -----------c) Sonographic Murphy's sign -------2) Cholescintigraphy (HIDA) scan -----------a) GOLD STD → but ONLY used when Abd US is inconclusive -----------b) DIAGNOSTIC FINDING → Failure to visualize gallbladder at 4-hrs post-injection or 30-mins post-morphine augmentation is -----b) TMT --------1) NPO, IV fluids, Pain meds (morphine) --------2) Empiric antibiotics ------------a) Community acquired ----------------1) Cephalosporins ------------b) Severe community acquired OR Cholecystitis in HIGH-RISK pts ----------------1) Piperacillin-Tazobactam ----------------2) Carbapenems ----------------3) Metro + Fluoroquinolone/cefepime ------------c) Health-care setting Acquired/Iatrogenic ----------------1) Vancomycin --------3) Laparoscopic cholecystectomy ------------a) W/in 48 hours of initiation of Antibiotics Cholangitis - Also presents w/ Fever, RUQ pain, Jaundice - Additionally, labs also show ↑ WBCs, ↑ Bilirubin, ↑ ALP, ↑ LFTS - However, key differences include: ---1) MISSING KEY SYMPTOMS of Cholangitis, including ------a) ALTERED MENTAL STATUS ------b) HYPOTENSION ---2) DOES NOT PRESENT W/: ------a) Murphys sign ----------[pt HAS Murphy sign] ---3) MGMT ------a) TMT ----------1) IV fluids + Antibiotics (BEST INITAL TMT) ----------2) IF CI to antibiotics --------------a) Decompression and drainage of biliary system -------------b) Endoscopic drainage -------------c) Percutaneous transhepatic biliary drainage (PTBD) -------------d) Surgical decompression if endoscopic procedure is unsuccessful

NBME CK 7 A 37F, w/ sickle cell disease presents d/t a 24-hrs Hx of fever, RUQ pain after eating, & nausea. Her last sickle cell crisis was 4 months ago. Her only med is folic acid. She is 5-ft, 6-in tall & weighs 145-lbs; BMI is 23. Temp is 100.8F, & pulse is 90/min. Exam shows mild scleral icterus. The abd is distended. Bowel sounds are decreased, & Murphy sign is present. Labs show: - Leukos----------------------------12K - BUN-------------------------------48 - Cr----------------------------------1.5 - Glucose---------------------------186 - Total bili---------------------------3 - ALP--------------------------------60 - Amylase---------------------------90 - Lipase-----------------------------40 (N 14-280) Abd US shows cholelithiasis, pericholecystic fluid, & a normal-sized CBD. Which of the following is the most likely Dx? (Acute cholecystitis VS Cholangitis)

ERCP - Pt has a Hx of gallstones in gallbladder (Cholelithiasis) & She recently had a cholecystectomy - However, she now presents w/ signs concerning for choledocholelithiasis (Gallstones in the CBD) -A) Signs/Symptoms ----1) Biliary colic -------a) RUQ pain ± waxing/waning ----2) Jaundice ----3) Light colored stools ----4) Dark urine ----5) Decreased appetite -B) MGMT ---a) Dx tests ------1) Transabdominal US (BEST INITIAL) ---b) TMT ------1) ERCP ------2) Elective cholecystectomy CT scan of abdomen - Would only use this if we were investigating GI cancer or if we suspected gallstone ileus - However, key differences include: ---1) PRESENTATION ------a) Gallstone presents w/ CLASSIC TRIAD ---------1) Pneumobilia ---------2) Presence of ectopic gallstone ---------3) SBO (complication) ------------[pt has NO GALLSTONE on US]

NBME CK 7 A 52F presents to the ED b/c of a 5-day Hx of continuous abd pain, nausea, & decreased appetite. 9 days ago, she underwent an elective laparoscopic cholecystectomy for symptomatic cholelithiasis. She was discharged from hospital 7 days ago. Temp Is 100.8F, Pulse is 108/min, BP is 102/64, RR are 20 min. Exam shows jaundice. Abd exam shows RUQ tenderness. Bowel sounds are presents through-out all quadrants. Labs show: - Hb--------------------------------12.2 - HCT-------------------------------36% - Leukos----------------------------17,2K - BUN-------------------------------48 - Cr----------------------------------1.5 - Glucose---------------------------186 - Total bili---------------------------7.2 - ALP--------------------------------251 - AST--------------------------------84 US of abd shows no fluid collections. Which of the following is the most appropriate next step in Dx? (ERCP VS CT scan of abdomen)

Vit B1 -A) Deficiency presents w/: ----1) Beri-Beri -------a) DRY ----------a.1) Muscle wasting ----------a.2) Neuropathy -------b) WET ----------b.1) Cardiomyopathy -----2) Wernicke-Korsakoff ---------a) Encephalopathy ------------a.1) Confusion ------------a.2) Ophthalmoplegia ------------a.3) Ataxia ---------b) Psychosis ------------b.1) Memory loss ------------b.2) Confabulation ------------b.3) Personality Vit B12 -A) Deficiency presents with: ----1) Ataxia ----2) Bilateral Paresthesias ----3) Impaired position/vibration sense ----4) SUBTLE NERUO SIGNS -------a) Depression/irritability -------b) Cognitive slowing -------c) Apparent dementia ----------[No ophthalmoplegia] Vit B2 (Riboflavin) -A) Deficiency presents w/: -----1) Cheilitis (Cracked angles of mouth) -----2) Glossitis (magenta colored tongue) -----3) Corneal vascularization

NBME CK 7 Two hours after admission to the hospital for detoxification, a 32M w/ alcoholism has the onset of ataxia & is unable to move either eye laterally. He has received IV dextrose & chlordiazepoxide therapy. Exam shows a wide-gait ataxia, bilateral rectus palsies, & horizontal nystagmus. A deficiency of which of the following is the most likely cause of his neurological symptoms? (Vit B1 VS Vit B12 VS Vit B2)

↓ Esophageal peristalsis ↑ LES tone

NBME CK 7 What are the expected findings for Achalasia? (Esophageal peristalsis---↑/↓ LES tone---------------↑/↓

Acalculous cholecystitis -A) RFs ----1) Critically ill & post-op patients ----2) Trauma/surgery ----3) Pts on TPN ----4) Multiple transfusions ----5) Sepsis -B) Signs/Symptoms -----1) Jaundice -----2) Fever -----3) Leukocytosis -----4) Pain Presentation → Dependent on ACUTE VS Chronic --------1) ACUTE cholecystitis -----------a) Pain is Constant, severe, & prolonged -----------b) Typically > 4-6 hours & located in RUQ/epigastric -----------c) ± radiation to shoulder/back --------2) CHRONIC cholecystitis ------------a) Pain is INTERMITTENT -----5) Biliary stasis → (↑ ALP & ↑ bilirubin) Cholecystolithiasis - Also Presents w/ Biliary colic (vomiting & nausea, diaphoresis, Dull RUQ/epigastric pain) after a fatty meal - Also may present w/ pain that radiates to shoulder - However key differences are: ---1) RF -----a) Acalculous cholecystitis are most common in MALES that are critically ill pts -------[Cholelithiasis typically presents in fat, forty year old females] ---2) CHOLELITHIASIS DOES NOT PRESENT w/ -----a) fever, -----b) leukocytosis, -----c) hypotension -----d) altered mental status --------[Cholelithiasis is an ACUTE condition caused by the presence of a stone]

One week after undergoing sigmoidectomy with end colostomy for complicated diverticulitis, a 67-year-old man has upper abdominal pain. During the surgery, he was transfused two units of packed red blood cells. His postoperative course was uncomplicated. Two days ago, he developed fever. He is currently receiving parenteral nutrition through a central venous catheter. He has type 2 diabetes mellitus, hypertension, and hypercholesterolemia. He is oriented to person, but not to place and time. Prior to admission, his medications included metformin, valsartan, aspirin, and atorvastatin. His temperature is 38.9°C (102.0°F), pulse is 120/min, and blood pressure is 100/60 mmHg. Examination shows jaundice of the conjunctivae. Abdominal examination shows tenderness to palpation in the right upper quadrant. There is no rebound tenderness or guarding; bowel sounds are hypoactive. Laboratory studies show: - Leukos----------------------------------------13,5K - Segmented neutros--------------------------75% - AST--------------------------------------------140 - ALT--------------------------------------------85 - ALP--------------------------------------------150 - Bilirubin (total)--------------------------------2.1 - Direct------------------------------------------1.3 - Amylase---------------------------------------20 Which of the following is the most likely diagnosis in this patient? (Acalculous cholecystitis OR Cholecystolithiasis)

IV tazobactam-piperacillin & percutaneous cholecystectomy - Pt most likely has acalculous cholecystitis -A) MGMT - MGMT of Acalculous cholecystitis is the SAME AS CHOLECYSTITIS: ---a) Dx tests ------1) US (BEST INITIAL TEST) ------2) HIDA (GOLD STD DIAGNOSTIC ---------a) However, mostly used if NO CHANGES are visualized on on US) ---b) TMT ------1) IV fluids, NPO ------2) Antibiotics (IV tazobactam-piperacillin) ------3) percutaneous cholecystectomy ----------a) W/in 24-72 hours after antibiotics ------4) ERCP w/ papillotomy ----------a) Used as a PALLIATIVE tmt in acalculus cholecystitis in pts w/ CI to Cholecystectomy

Three days after undergoing coronary artery bypass surgery, a 72-year-old man has severe right upper quadrant pain, fever, nausea, and vomiting. He has type 2 diabetes mellitus, benign prostatic hyperplasia, peripheral vascular disease, and chronic mesenteric ischemia. He had smoked one pack of cigarettes daily for 30 years but quit 10 years ago. He drinks 8 cans of beer a week. His preoperative medications include metformin, aspirin, simvastatin, and finasteride. His temperature is 38.9°C (102°F), pulse is 102/min, respirations are 18/min, and blood pressure is 110/60 mmHg. Auscultation of the lungs shows bilateral inspiratory crackles. Cardiac examination shows no murmurs, rubs or gallops. Abdominal examination shows soft abdomen with tenderness and sudden inspiratory arrest upon palpation in the right upper quadrant. There is no rebound tenderness or guarding. Laboratory studies show the following: - Hb---------------------------------------------13.1 - Hct---------------------------------------------42% - Leukos-----------------------------------------15,7K - Segmented neutrophils-----------------------65% - Bands-------------------------------------------10% - Lymphos---------------------------------------20% - Monocytes-------------------------------------3% - Eosinos-----------------------------------------1% - Basophils--------------------------------------0.5% - AST----------------------------------------------40 - ALT----------------------------------------------100 - ALP----------------------------------------------85 - Bilirubin total------------------------------------1.5 - Direct--------------------------------------------0.9 - Amylase------------------------------------------90 Abdominal ultrasonography shows a distended gallbladder, thickened gallbladder wall with pericholecystic fluid, and no stones. Which of the following is the most appropriate next step in management? (ERCP w/ papillotomy OR IV tazobactam-piperacillin & percutaneous cholecystectomy)

Cholelithiasis - Occurs when stones precipitate in Bile - Presentation includes: 1) RF's ---a) Fat, Fertile, 40 year old, Female ---b) Crohns, CF, fibrates, estrogen therapy, rapid wt loss, cirrhosis, Native American, TPN, ileal resection 2) Signs ---a) Biliary colic (sudden intermittent pain after meal) ± Vomiting & Chills 3) Complications ---a) Cholecystitis ---b) Cholangitis Cholecystitis --Inflammation of the gallbladder → Usually as a complication of cholelithiasis 1) Presentation ---a) Cholecystitis & cholelithiasis are essentially the same so present the same → However, Cholecystitis will have ABNORMAL LABS (KEY DIFFERENTIATION), including: -----1) ↑ WBC count -----2) ↑ ALP Diagnostic --1) US --2) HIDA Choledocholilithiasis - Subset of cholelithiasis is which GALLSTONES ARE FOUND IN CBD 1) Presentation ----a) Biliary colic (sudden intermittent pain after meal) ± Vomiting Chills ----b) KEY DIFFERENTIATING SYMPTOMS (from cholelithiasis & cholecystitis) --------1) Pale stools --------2) Jaundice --------3) Dark urine 2) Diagnostic -----1) US -----2) MRCP (Intermediate risk pts) ----------For pts w/ No visualized stone, No high risk features (dilated CBD, no ↑ bilirubin, no cholangitis) -----3) ERCP (High-risk pts) 3) TMT ----1) ERCP (diagnostic & TMT) ----2) Laparoscopic cholecystectomy (to prevent future recurrences) Acute cholangitis - Ascending bacterial infection from CBD (as a complication to choledocholilithiasis 1) Presentation ---a) Biliary colic (RUQ pain) + Fever + Jaundice ---b) KEY DIFFERENTIATION -------1) Hypotension -------2) Altered mental status 2) Labs ---a) ↑ WBC's & ↑ ALP ---b) KEY DIFFERENTIATION -------1) ↑ Bilirubin -------2) ↑ LFT's (↑AST, ↑ALT) 3) MGMT ---1) Fluids & IV antibiotics ---2) Decompression & endoscopic drainage of biliary system ---3) Percutaneous trans-hepatic biliary drainage (PTBD) ---4) Surgical decompression (if endoscopy fails)

What is the difference between choledocholelithiasis, cholecystitis, Acute cholangitis, cholecystolithiasis?


Conjuntos de estudio relacionados

IS435 Daeeun Daniel Choi- midterm

View Set

Income Statement, Balance Sheet, Cash Flow Statement

View Set

Which bone articulates with what?

View Set

Chapter 7: The Rise and Spread of Islam questions

View Set

Physical Science Semester 1 Study Set

View Set